MBE Evidence

Ace your homework & exams now with Quizwiz!

Can woman bring in statement by father before he died recognizing her as his child?

Even though it is hearsay, it will be admitted as a statement of pedigree. Discussion of correct answer:FRE 804(b)(4) is commonly known as the pedigree hearsay exception, and it applies to the following statements of personal or family history made by an unavailable declarant: "(A) A statement concerning the declarant's own birth, adoption, marriage, divorce, legitimacy, relationship by blood, adoption, or marriage, ancestry, or other similar fact of personal or family history, even though the declarant had not means of acquiring personal knowledge of the matter stated; or (B) a statement concerning the foregoing matters, and death also, of another person, if the declarant was related to the other by blood, adoption, or marriage or was so intimately associated with the other's family as to be likely to have accurate information concerning the matter declared." The man, the unavailable declarant who is now deceased, was the woman's biological father. His out-of-court statement, "Here's my girl," is being offered by the woman substantively to prove her status as an heir and claim a share of his estate. Thus, this answer choice is correct. Regarding answer choice (B), note that the family record hearsay exception under FRE 803(13) applies to the statements (or written records) made in family Bibles, inscriptions on family portraits, and engravings on tombstones.

City employee takes case to Federal Court. What law governs?

Federal common law. Discussion of correct answer:Federal Rule of Evidence 501 states that "...the privilege of a witness...shall be governed by the principles of the common law as interpreted by the courts of the United States in the light of reason and experience." In other words, on matters governed by federal substantive law--and this will generally be true in criminal cases brought by the United States and private federal question cases--federal courts are to apply and develop a privilege. Because this question deals with a federal question (namely whether there is a violation of the Civil Rights Act), the issues should be resolved under federal privilege law. In this type of action, determinations of privilege are based on the federal courts' interpretation of common law. Federal Rule of Evidence 501 states that "...the privilege of a witness...shall be governed by the principles of the common law as interpreted by the courts of the United States in the light of reason and experience."

Trial 1: truck driver's negligence. Trial 2: Child due med pay. Is child's injury admissible at Trial 1? Motion to suppress it?

Grant the motion, because evidence of the child's injuries is not relevant. Discussion of correct answer:Irrelevant evidence is not admissible. Evidence is relevant if it has a tendency to make some fact at issue more or less likely; this is known as logical relevancy. Under the court's order to bifurcate the trial, damages are not an issue in the first trial; the jury in the first trial is only determining whether the driver was negligent. Therefore, the child's injuries are not relevant to any matter of consequence in the first trial. The motion in limine to exclude evidence of the child's injuries should be granted because the evidence is not logically relevant. Concerns Rule 403 balancing test to determine legal relevance. Relevant evidence may be excluded if the probative value of the evidence is substantially outweighed by the danger of unfair prejudice, confusing the issues, misleading the jury, undue delay, wasting time, or needlessly presenting cumulative evidence. In other words, for relevant evidence to be excluded under the Rule 403 balancing test, the probative value must be substantially less than the danger of misuse. The probative value just being "less than" the danger of misuse, as stated in this answer choice, is insufficient for exclusion under the Rule 403 balancing test. This is also not the best answer choice because the Rule 403 balancing test applies only to relevant evidence. If evidence is not logically relevant, then it is unnecessary to consider legal relevance. Evidence is logically relevant if it has some tenancy to make a fact of consequence more or less likely. Because damages are not being determined in the first trial, the child's injuries do not make any fact at issue in the first trial more or less likely, and the evidence is therefore not relevant.

Woman mailed letter to company accepting offer on Jan 18. What is true?

If the company produces some evidence to the contrary, the court should not instruct the jury on the presumption. Discussion of correct answer:Under Federal Rule of Evidence 301, most presumptions can be rebutted by evidence that contradicts the presumption. If one party lays a proper foundation for the presumption and the other party does not introduce any contrary evidence, the judge will instruct the jury on the presumption. If the other party produces even a scintilla of evidence that is credible which contradicts the presumption, the judge will not instruct the jury on the presumption. This is known as the "bursting bubble" approach. The woman says she mailed the letter to the company. The company, on the other, hand produces some evidence to the contrary, so they have met their burden of producing contradictory evidence. The "bubble" has burst, and the judge should not instruct the jury about the presumption. Incorrect. No matter what the company testifies to, the court should instruct the jury that they may presume the letter was received. Under Federal Rule of Evidence 301, most presumptions can be rebutted by evidence that contradicts the presumption. Since the company produced evidence, the presumption is contradicted and thus rebutted.

Spousal vs Marital Priviledge

In federal courts and in most states, there are two recognized marital privileges. First, the spousal testimonial privilege, which allows a witness-spouse to refuse to testify against his/her spouse in a criminal (but not civil) proceeding. However, the spouse-witness may voluntarily testify against his/her spouse as to nonconfidential communications made during the marriage [Trammel v. United States, 445 U.S. 40 (1980)]. The testimony privilege only applies while the spouses are presently married to each other, but it extends to pretrial matters as well. Former spouses may not assert the testimony privilege or prevent former spouses from testifying. The second privilege is the marital communications privilege, which protects confidential communications made during the existence of a valid marriage intended by the spouses to remain confidential. The communications privilege applies in both criminal and civil proceedings, and either spouse may invoke the privilege. However, there are three recognized exceptions to the communications privilege: (1) suits between the spouses (including divorce and child custody disputes); (2) suits where one spouse is charged with a crime or tort against the other or the children in the household; and (3) where the spouses jointly participate in criminal acts--i.e., the joint participant exception.

Offering someone you hit money and med pay to not sue her at the scene - admissible?

Inadmissible, as a statement made in connection with settlement negotiations. fascinating... Discussion of correct answer:You must read the facts carefully here to see that this is an offer to fully settle the claim (offer to compromise) and not just an offer to pay medical bills. Under Federal Rule of Evidence 408, an offer to compromise a claim disputed as to either validity or amount, or an acceptance of such an offer, may not be introduced as evidence that the disputed claim is either valid or invalid or as evidence as to its proper amount. Conduct or statements made during compromise negotiations are also barred from evidence under Federal Rule of Evidence 408. There is a public policy to encourage settlement. Therefore, the housewife's statement regarding her inattentiveness would be barred from evidence because it was made in conjunction with her offer to settle any claim for $5,000. Remember, MBE questions are tricky; look for phrases such as "release," "promise not to sue," or "let's end this thing," which indicate an offer to settle rather than just an offer to pay medical expenses.

Teller selected defendant in lineup and then moved out of town. Can police testify as to choosing?

Inadmissible, as hearsay not within any exception. Discussion of correct answer:This looks at first as if it should be admissible under the hearsay exception for prior identifications. Rule 801(d)(1)(C) does classify a prior identification of a person made after perceiving that person as nonhearsay. Thus, a witness may testify that she earlier identified a criminal defendant. However, the declarant must testify at trial and be subject to cross-examination. Here, the teller did not testify. Therefore, the police officer's testimony regarding the teller's identification would be hearsay and is not within any exception. Incorrect. Admissible, as a prior identification. Although this is a tempting choice, it is wrong. The prior identification testimony fails because the declarant is not in court and subject to cross-examination. This shows the need for a careful factual reading.

How a presumption shifts the burden of going forward to the opposing party

It creates a presumption that notice was received, but the presumption is lost if the employee provides contrary evidence that rebuts the presumption. Discussion of correct answer:A party has a burden of persuasion to establish a claim or defense and faces a directed verdict on that issue if they fail to provide the required proof. A presumption shifts the burden of going forward to the opposing party by allowing an inference that can be drawn based on a set of facts (basic facts) that show a high probability of the existence of another set of facts (presumed facts). This presumption can be rebutted with contrary evidence. If the opposing party provides such contrary evidence to rebut the presumption, the presumption is lost and the burden of persuasion shifts back to the original party. In this case, the employer bears the burden of persuasion with regard to proving its defense to the employee's breach of contract claim, i.e. that it provided proper notice under the contract. The employer has a date stamped receipt proving mailing of the notice and the employment contract indicates notice is presumed received if it has been mailed. Therefore the employer can assert the presumption that the notice was received. If the employee provides contrary evidence that rebuts the presumption that the notice was successfully mailed, however, the presumption is lost.

Court issues ruling deciding case that attorney disagrees with. What must attorney do to keep case for appeal?

Make a formal offer of proof at the time the objection is made, unless the substance of the evidence was apparent from the context. Discussion of correct answer:To preserve an error related to the improper exclusion of evidence, the trial attorney must make a formal offer of proof at the time of the objection unless the substance of the evidence was apparent from the context in which questions were asked. An offer of proof involves demonstrating to the court the substance of the evidence that the proponent would introduce if permitted. Absent such an offer of proof, the party loses the right to appeal based upon the erroneous exclusion of evidence. Incorrect. State for the record that the judge has made an error by excluding the testimony. Merely stating for the record that the judge has made an error is not sufficient to preserve the record for review, if the substance of the excluded evidence is not apparent from the context in which questions were asked. To preserve the right to appeal a ruling that resulted in the exclusion of evidence, the party seeking to have the evidence admitted must make an offer of proof so that the court is aware of the substance of the evidence at issue, unless the substance of the evidence is apparent from the context within which the questions were asked.

Which of the following convictions would be most admissible against the father who punched the referee?

Misdemeanor fraud and sentenced to sixty days in jail seven years ago. Discussion of correct answer:F.R.E. 609(a) provides: "For the purpose of attacking the credibility of a witness, evidence that he has been convicted of a crime shall be admitted if elicited from him or established by public record during cross-examination but only if the crime (1) was punishable by death or imprisonment in excess of one year under the law under which he was convicted, and the court determines that the probative value of admitting this evidence outweighs its prejudicial effect to the defendant, or (2) involved dishonesty or false statement, regardless of the punishment." F.R.E. 609(b) provides: "Evidence of a conviction under this rule is not admissible if a period of more than ten years has elapsed since the date of the conviction, unless the court determines in the interests of justice, that the probative value of the conviction supported by specific facts and circumstances substantially outweighs its prejudicial effect." This question is testing on both parts of F.R.E. 609. Choice (A) is incorrect because it is unlikely a 14-year-old felony theft conviction would be highly probative in a civil battery case as it is too remote. Choice (B) is incorrect because F.R.E. 609(a)(1) applies only to felonies, not misdemeanors, and under section (2) only misdemeanors involving dishonesty or false statement are admitted. Battery definitely does not fit within this class. Choice (C) is incorrect because even though the assault conviction is a felony, it will probably be inadmissible under the 10-year limitation under F.R.E. 609(b), subject to the discretion of the court. Choice (D) is correct because in this case the conviction involves a crime of dishonesty and it is within the 10-year rule.

Def on trial for murder. Prosecution brings in witness attesting to times def lied. Admissible?

No, as improper impeachment. Discussion of correct answer:This is a difficult evidence question that exemplifies the interplay of impeachment and character evidence, a favorite subject of the bar examiners. Here, the defendant has made a proper offer of character evidence through the friend. The prosecutor has at least two choices: impeach the friend; and/or offer rebuttal character evidence showing Defendant to be a violent man. Here, the prosecutor chose the first path. Under FRE 608(a), testimony can be offered in the form of opinion or reputation concerning the friend's character for truthfulness. However, this evidence is admissible only for the purpose of impeachment. The prosecutor attempted the impeachment of the credibility of the friend, but not through opinion or reputation evidence. Instead, she attempts to do so with extrinsic evidence of prior instances of untruthful conduct by the friend. This she cannot do. Incorrect. No, as improper rebuttal of character evidence. This choice is incorrect, because the prosecutor is not offering rebuttal character evidence to show the defendant to be a violent man.

Victim wrote out description of defendant, then died. May description be introduced?

No, because it is hearsay not within an exception. Discussion of correct answer:Under Federal Rule of Evidence 801(d)(1)(C), where a witness has made a previous out-of-court identification of a person after perceiving that person, the prior identification statement is not hearsay, even though offered to prove the truth of the matter asserted (that is, the identification). The prior identification statement will be admissible as substantive evidence. However, the admission of a prior identification is permitted only where the declarant is available to testify at the trial and will be subject to cross-examination. Here, it is likely that the victim's written statement would be regarded as a prior identification. However, because the victim is in a coma and not available to testify at trial, the victim's previous written description cannot be admitted into evidence. Incorrect. Yes, because it is a prior identification. The testimony of a percipient witness that the criminal defendant is the same person whom the witness observed committing a crime, or a description of the person whom the witness observed committing the crime, would clearly be admissible as relevant evidence in the prosecution of that defendant. Here, however, the testimony is a hearsay declaration containing a description of the perpetrator. Unless one of the exceptions to the hearsay rule applies, the evidence is inadmissible, even though it may be relevant to the issue of identification.

Letter from defendant's fiancee showed intent to be at location at the time. Is this admissible against him by plaintiff?

No, because it is hearsay not within any exception. Discussion of correct answer:Federal Rule of Evidence 803(3) provides that a statement regarding the declarant's state of mind at the time of the statement is admissible as an exception to the hearsay rule. Pursuant to this rule, a written statement expressing a declarant's state of mind may be admitted to show his intent at the time of the writing [Mutual Life Insurance Co. v. Hillmon, 145 U.S. 285 (1892)]. However, such a statement of intent is only admissible against the declarant, and not someone else. As such, the letter would not be admissible against the defendant to prove his intent to be in Hawaii, because he is not the declarant. As such, it would be hearsay not within any exception, and therefore inadmissible. Incorrect. Yes, because it represents a statement of the fiancee's intent. Federal Rule of Evidence 803(3) provides that a statement regarding the declarant's state of mind (such as intent) at the time of the statement is admissible as an exception to the hearsay rule. Pursuant to this rule, a written statement expressing the declarant's state of mind may be admitted to show his intent at the time of the writing [Mutual Life Insurance Co. v. Hillmon, 145 U.S. 285 (1892)]. However, such evidence is only admissible to prove the declarant's state of mind, not that of someone else. Here, the defendant's fiancee's intent to be with the defendant in Hawaii on July 4 is clearly shown by the letter. Because the letter was written by the defendant's fiancee, and not by the defendant, it cannot be admitted to prove that the defendant was in Hawaii at that time, because the defendant was not the declarant.

Place burgled, guard wrote down description of perp. guard died. Is written description admissible?

No, because it is hearsay not within any exception. Discussion of correct answer:In Crawford v. Washington [541 U.S. 36 (2004)], the Supreme Court stated that, in accordance with the intent of the Sixth Amendment Confrontation Clause, prior "testimonial" evidence is inadmissible unless: (1) the declarant is unavailable; and (2) the defendant had a prior opportunity to cross-examine the declarant. Actual cross-examination is not required, but the opportunity to do so, such as at a deposition, is required. Because the night guard died before the defendant was arrested for the crime, the defendant did not have the opportunity to cross-examine the night guard. Therefore, the night guard's statement, even though he is an unavailable witness, should be excluded. Incorrect. Yes, because it is a past recollection recorded. Federal Rule of Evidence 803(5) allows for evidence in the form of a writing to be admitted into evidence if the witness has a personal knowledge of the matter but cannot now recall it. This is the "recorded recollection" hearsay exception. However, that does not apply here because the night guard is dead and therefore cannot establish that the writing reflected his views at the time.

Witness called to testify as to statement made to police which police wrote down. Can it be admitted?

No, because it is hearsay not within any exception. Discussion of correct answer:The woman's statement was made out of court and is now being offered for the truth of the matter asserted therein (i.e., that the man had run a red light and swerved into her car). The statement is therefore hearsay not within any applicable exception. Thus, the statement is inadmissible. Incorrect. Yes, because it constitutes a prior consistent statement. Under Federal Rule of Evidence 801(d)(1)(B), a declarant's prior statement that is consistent with her testimony is not hearsay only if it is offered to rebut a charge of recent fabrication or improper influence or motive. In addition, the declarant must testify and be subject to cross-examination concerning the statement. If these conditions are met, the statement may be used as substantive evidence. Here, there is no indication of a charge of recent fabrication or improper influence or motive. And given that the testimony is being offered during the direct examination of the woman, it is not possible that the opposing party could yet have raised such an issue. As such, the statement is inadmissible.

Defendant seeks to testify that she told police she didn't know where *that* warehouse was. You know, the one were *she* held 100 dogs in squalor. Is is available for testimony?

No, because it is hearsay not within any exception. Discussion of correct answer:Under Federal Rule of Evidence 801(d)(1)(B), a declarant's prior statement that is consistent with her testimony is not hearsay only if offered to rebut a charge of recent fabrication or improper influence or motive. In addition, the declarant must testify and be subject to cross-examination concerning the statement. If these conditions are met, the statement may be used as substantive evidence. Here, the woman's statement to the arresting officer is an out-of-court declaration offered for the truth of its assertion. The statement is relevant to her guilt or innocence only if it is true that she did not know the location of the warehouse. According to the facts, there is nothing to show that the prosecution attacked her credibility in a way that she had recently fabricated her trial testimony, or that an improper motive or influence to testify falsely arose after the making of the prior consistent statement. She was testifying to her out-of-court statement during direct examination. Without such foundational circumstances, the prior inconsistent statement is inadmissible hearsay. Incorrect. Yes, because she is testifying in court and is subject to cross-examination. The form of the question is improper, given that the woman was testifying to her out-of-court statement during direct examination. The facts do not indicate that she had been impeached in a way that suggested recent fabrication or motive to falsify arising after a prior inconsistent statement. As such, the fact that the woman is subject to cross-examination does not make her statement admissible at this point in her testimony.

Statement to doctor by plaintiff that his hand had cleared up from the incident. Admissible Doctor Testimony?

No, because it is hearsay not within any exception. Discussion of correct answer:Under Federal Rule of Evidence 803(4), hearsay statements made for the purpose of medical diagnosis or treatment, describing medical history or past or present symptoms, pain, or sensations are admissible as an exception to the hearsay rule. To be admissible under this exception, the statement must be made to a physician or other person who is assisting in the treatment or diagnosis (e.g., a paramedic, nurse, hospital receptionist) of the declarant/patient. Statements regarding the cause of injury will be admissible under this exception "insofar as reasonably pertinent to diagnosis or treatment." However, where the circumstances are such that the guarantees of reliability normally associated with this exception are not present, such a statement may be excluded. Here, the cook went to the doctor for an annual checkup rather than for treatment for his hand. At the time of his checkup, the cook knew that the trial against the grocery store was pending, and he would therefore have a strong motive to manufacture evidence. In addition, as he was not being treated by the doctor for a hand injury, his statement that his hand problems had cleared up prior to his fall at the grocery store would not serve to aid the doctor in his treatment of the cook. As such, under the circumstances, this exception to the hearsay rule does not apply, and the cook's statement is inadmissible. Incorrect. No, because the cook's statement was made in relation to a past physical condition. There is no blanket exception to the hearsay rule for statements made in relation to a past physical condition. Hearsay declarations describing then-existing mental, emotional, or physical condition are admissible under Federal Rule of Evidence 803(3). Under Federal Rule of Evidence 803(4), hearsay statements describing medical history or past or present symptoms, pain, or sensations are likewise excepted from the hearsay rule if made for the purpose of treatment or diagnosis. However, because the cook's statement regarding his condition prior to the accident was not a description of his then-existing physical condition, nor was it made for the purpose of treatment or diagnosis, it would not fall within any exception to the hearsay rule.

Sis on trial for attacking sis. Brings in witness to say she was a good neighbor. Admissible?

No, because it is not directed toward a pertinent trait of the defendant's character. Discussion of correct answer:A defendant in a criminal case may offer character evidence on his own behalf, but the evidence here doesn't qualify. Federal Rule of Evidence 404 provides that evidence of a person's character or a trait of his character is not admissible for the purpose of proving that he acted in conformity therewith on a particular occasion, except when evidence of a pertinent trait of his character is offered by the accused, or by the prosecution to rebut the same. The witness's testimony that the defendant is a "good solid neighbor" is not admissible because it is not a pertinent trait of her character. Here, the trait of being peaceful would be a pertinent trait, but being a good neighbor is not. Incorrect. Yes, because it is relevant to show the defendant's character in this case. This choice is incorrect. Usually a criminal defendant may offer character evidence on his own behalf, but it must be relevant to the crime of which he is accused. For instance, if the witness had testified to the defendant's reputation for peacefulness in the community, it would probably have been admitted.

Defense witness testifies Defendant was a good neighbor. Admissible? Man hit his bro with a baseball bat.

No, because it is not directed toward a pertinent trait of the man's character. Discussion of correct answer:A defendant in a criminal case may offer character evidence on his own behalf, but the evidence here doesn't qualify. Federal Rule of Evidence 404 provides that evidence of a person's character or a trait of his character is not admissible for the purpose of proving that he acted in conformity therewith on a particular occasion, except when evidence of a pertinent trait of his character is offered by the accused, or by the prosecution to rebut the same. The witness's testimony that the man is a "good neighbor" is not admissible because it is not a pertinent trait of his character. Here, the trait of being peaceful would be a pertinent trait, but being a good neighbor would not. Incorrect. Yes, because it is relevant to show the man's character. Usually, a criminal defendant may offer character evidence on his own behalf, but it must be relevant to the crime of which he is accused. For instance, if the witness had testified to the man's reputation for peacefulness in the community, it would probably have been admitted.

Def on trial for rape, woman had called 911 on night of alleged rape. Def offers witness who will say woman is promiscuous. Allowed?

No, because it violates the rape shield law. Discussion of correct answer:In a criminal case, barring certain limited exceptions, evidence of a rape victim's prior sexual conduct or sexual predisposition is inadmissible. Pursuant to these exceptions, such evidence is admissible to prove that a person other than the accused is the source of semen, the victim's injury, or other physical evidence, or if exclusion of the evidence would violate the constitutional rights of the defendant. Specific instances of sexual behavior between the accused and the victim may also be offered to prove consent. Because none of these exceptions apply to the proposed testimony, it is inadmissible. Incorrect. No, because the evidence is unfairly prejudicial. The Federal Rules of Evidence specifically address the issue of when evidence of the prior sexual conduct of a victim of a sex crime is admissible. Pursuant to the "rape shield law," barring certain limited exceptions, evidence of a rape victim's prior sexual conduct is generally inadmissible. Thus, while it may be true that the witness's testimony might be unfairly prejudicial, the better analysis of the admissibility of this evidence would take into account the particular provisions of the rape shield law.

Lay Opinion Testimony

No, because lay opinion is admissible concerning the value of land owned by the lay opinion witness. Discussion of correct answer:Under the Federal Rules of Evidence, lay opinion testimony is permissible if: a) rationally based on the perception of the witness; and b) helpful to a clear understanding of the testimony or his testimony or the determination of a fact in issue. Lay opinion testimony may be given by an owner of land concerning the value of that land because such an opinion is rationally based on lay perception and does not require the qualifications of an expert to interpret.

Plaintiff seeks to testify about phone conversation where he accepted contract from Defendant. Defendant denies. Is this allowed?

No, because of lack of foundation. Discussion of correct answer:This type of testimony is inadmissible unless a proper foundation is laid. The cases are in agreement that a mere assertion of identity by a person talking on the telephone is not sufficient evidence of the authenticity of the conversation. Some additional evidence is required to lay a foundation, such as the fact that the speaker discloses knowledge of facts known peculiarly to him. That would not help the plaintiff here because the facts indicate the plaintiff sent similar offers to all of the defendant's neighbors. For instance, it would have been a sufficient foundation to authenticate the phone call if the plaintiff had testified as to more occasions when he heard the defendant's voice to enable him to be familiar enough to give a lay opinion on this matter. The facts are devoid of such evidence. Therefore, the evidence is probably inadmissible because it lacks foundation. Incorrect. No, because this is inadmissible hearsay. This incorrectly focuses on hearsay as the reason for exclusion. This would be non-hearsay because an acceptance in a contract case is a "verbal act" and is treated as non-hearsay. The problem here is that the information given does not show that the plaintiff laid a sufficient foundation to show that it was actually the defendant with whom he spoke on the phone, so it is inadmissible for that reason.

Truck driver wishes to testify he did *everything* to avoid this lawsuit, even offering to settle. ($500...). Is this admissible?

No, because offers to compromise are not admissible. Discussion of correct answer:FRE 408 makes offers to compromise disputed claims inadmissible on the issue of liability or damages. This is true even where, as here, the party that made the offer is attempting to admit it at trial in mitigation. Incorrect. No, because offers to pay medical bills are not admissible. This answer is incorrect because it refers to Rule 409, which does not apply here. The driver of the truck's offer was intended as an offer to settle the entire matter, not just an offer to pay medical bills.

Witness claims man was talking to himself and, in his opinion, was schizophrenic. Admissible?

No, because schizophrenia is a legal conclusion. Discussion of correct answer:Pursuant to FRE 701, if the witness is not testifying as an expert, his testimony in the form of opinions or inferences is limited to those opinions or inferences which are rationally based on the perception of the witness and helpful to a clear understanding of his testimony or the determination of a fact in issue. The testimony must not be based on scientific, technical, or other specialized knowledge. In this question, the facts are silent as to the witness's background or education. Therefore, his testimony is limited to being a lay witness. The facts state that the witness witnessed the man's will. Thus, his perceptions as to the man's mental state would assist the trier of fact in determining the man's competency to make a will. The proper scope of non-expert opinion includes such perceptions as speed and other measurements; physical states such as intoxication or injury, and sanity of the testator. Where the sanity of the testator is at issue, the opinion may be given by a subscribing witness. Legal conclusions, however, must be avoided. For example, the lay witness cannot testify that an individual was schizophrenic, alcoholic, or that an accident victim fractured his spine. Here, the facts state that the witness was a subscribing witness to the man's will. However, the witness's testimony contained a legal conclusion that the man was schizophrenic. Therefore, his testimony to that point is inadmissible. Incorrect. No, because sanity is beyond the scope of lay opinion. The proper scope of non-expert opinion includes such perceptions as speed and other measurements; physical states such as intoxication or injury, and sanity of the testator (given by a subscribing witness). Hence a lay witness can testify as to a testator's sanity, however, he cannot make legal conclusions such as whether a person is schizophrenic.

Sustain objection to questions on fraudulent business practices by prosecution against defendant businessman charged with arson?

No, because the conviction can be used to show that the realtor has the propensity to lie. Discussion of correct answer:The Federal Rules of Evidence allow any witness to be impeached with evidence that he or she has been convicted of a crime (felony or misdemeanor) involving dishonesty (such as embezzlement, fraud, perjury). Fraudulent business practices include the notion of dishonesty because fraud involves false statements or omissions that have the effect of false statements, so conviction for having engaged in such practices would qualify as a crime of dishonesty. Therefore, this answer is correct. Incorrect. Yes, because the conviction of fraudulent business practices is unrelated to a charge of arson. The conviction is admissible because it involved dishonesty. It is, thus, circumstantially relevant to the realtor's character for truth. For this purpose, it is immaterial whether the prior conviction offered into evidence for impeachment purposes has any similarity to the charged crime. The lack of similarity is unimportant and cannot serve as a basis upon which to exclude otherwise admissible evidence.

Two expert, both state in their opinion the defendant did/did not do it. Admissible?

No, because the defense witness made a determination reserved for the trier of fact. Discussion of correct answer:Federal Rule of Evidence 704 provides that a witness may not provide testimony in the form of an opinion or inference as to an ultimate issue to be decided by the trier of fact. Expert testimony is meant to help inform the judgment of the trier of fact, not replace that judgment. Expert testimony will not be objectionable when it merely touches on the ultimate issue, but it will be inadmissible where, as occurred here, the expert witness states that the defendant was insane when that is the ultimate question placed before the jury. By stating that the defendant was insane at the time that the crimes were committed and that the defendant would have been unable to formulate the necessary intent to commit the crimes, the defendant's expert witness exceeded the scope of permissible testimony and the testimony was improper. Incorrect. Yes, because both witnesses offered their conclusions as opinions rather than irrebuttable presumptions. It is not sufficient that an expert state that the expert's conclusion is "in their opinion" when opining as to the ultimate issue, which is to be decided by the trier of fact. The fact that the expert implies that the jury may disagree with the opinion is of no consequence. The rule is that an expert witness cannot give an opinion on the ultimate issue. Even if the expert qualifies her answer, the risk remains that the jury will improperly believe that they defer to the expert and allow the expert's judgment to be substituted for their own.

Defendant said she had passed red light during an offer to settle. May plaintiff use this?

No, because the driver's statement cannot be severed from the offer to settle. Discussion of correct answer:Evidence of an offer to settle a claim is inadmissible to prove the validity of a disputed claim. Statements and admissions of fault made with the offer to settle the claim are also not admissible, even to impeach the witness by a prior inconsistent statement. Here, the driver offered to pay the bicyclist for the damage to the bike in order to settle the claim. The driver's statement that she was late to work and didn't stop at the red light were made in connection with her offer to settle. Because she never paid the bicyclist anything, and the parties had a disagreement over the value of the claim, there is now a dispute, and therefore the statements are inadmissible at trial to prove that the driver was at fault. Incorrect. Yes, because the driver's statement is a statement against interest. This answer is incorrect because it reaches an inaccurate conclusion by stating an irrelevant rule of law. Hearsay, which is an out-of-court statement offered to prove the truth of the matter asserted, is inadmissible unless an exemption or an exception applies. If the declarant is unavailable, an exception to the hearsay rule applies where the statement is against the declarant's interest. The statement must be one that a reasonable person would not have made unless the person believed the statement to be true. This hearsay exception is not applicable because the driver is not unavailable. Further, the declarant - the driver - is the party-opponent, and a statement is not hearsay when the statement is offered against the opposing party, and the statement was made by the opposing party. Therefore, the statement is not hearsay, but it is not admissible because it was made in connection with an offer to settle the dispute.

Prosecution testimony: man's cousin asked him "Yo, where you get that watch? You must have stole it" and the man was silent and did't respond. Is this admissible?

No, because the man had no reason to respond to his cousin's inquiry. Discussion of correct answer:The fact that a party failed to respond (by statement or act) in the face of a provocative accusation or event may under certain circumstances be classified as a statement by silence. The party's failure to respond will be considered a statement if: 1) the party against whom the evidence is offered heard, understood and was capable of responding to the statement; and 2) a reasonable person in the party's position would have responded (e.g., denied the accusation). Here, a reasonable person in the man's position would have been likely to interpret the comment as a joke. Therefore, even if he heard and understood the comment, the man had no reason to respond. Thus, a denial was not required, and his silence cannot be taken as a statement. On the MBE, look for questions in which someone is accused of a malum in se crime like rape or murder, for which a normal person would automatically deny culpability. Incorrect. Yes, as a statement by a party-opponent. The fact that a party failed to respond (by statement or act) in the face of a provocative accusation or event will be constitute a statement only if: 1) the party against whom the evidence is offered heard, understood and was capable of responding to the statement; and 2) a reasonable person in the party's position would have responded (e.g., denied the accusation). Here, a reasonable person in the man's position would have been likely to interpret the comment as a joke. Therefore, even if he heard and understood the comment, the man had no reason to respond. As such, his silence cannot be taken as a statement.

Officer claims she still had the defendant license which she forgot to give back after examining, Def was 2 blocks from crime scene at the time. Can she testify as to this?

No, because the original writing rule requires the license to be produced into evidence. Discussion of correct answer:According to the best evidence rule, sometimes referred to as the original writing rule, in proving the terms of a writing, where the terms are material, the original writing must be produced unless it is shown to be unavailable for some reason other than the serious fault of the proponent. In the present example, the name on the license is in issue. The officer is proposing to testify that the name on the license was that of the defendant. Here, the contents of the license (which is the writing relied upon for purposes of the best evidence rule) are in issue. As a result, choice (C) is correct because the license itself must be produced into evidence as the "best evidence" of the terms or contents contained therein. Note choice (B) is wrong because the officer is, in fact, testifying to the contents of the license. Therefore, the name on the license is in issue to determine the identity of the driver of the vehicle. Incorrect. Yes, because the identity of the driver, not the contents of a writing, is at issue.

Psychologist who examined defendant brought in to outline defendant's psychopathy by Prosecutor. Admissible?

No, because the probative value of the evidence is substantially outweighed by the danger of unfair prejudice. Discussion of correct answer:To be admissible, evidence must be relevant. Federal Rules of Evidence 401 states that evidence is relevant if it tends to make a material fact more probable or less probable than it would be without the evidence. However, in certain cases, relevant evidence may be excluded for other reasons. For example, under Federal Rules of Evidence 403, a judge may exclude relevant evidence if she finds that the probative value of the evidence "is substantially outweighed by the danger of unfair prejudice, confusion of the issues, misleading the jury, or by considerations of undue delay, waste of time or needless presentation of cumulative evidence." Here, it is quite likely that the probative value of the psychiatrist's described testimony is substantially outweighed by the danger of unfair prejudice. Specifically, admitting the psychiatrist's testimony would create the danger that the jury would rely upon the psychological profile and overlook other evidence tending to prove or disprove the defendant's guilt. Under Federal Rules of Evidence 401, evidence is relevant if it tends to make a material fact more probable or less probable than it would be without the evidence. Here, it is true that the psychiatrist's testimony is relevant. However, in certain cases, relevant evidence may be excluded for other reasons. Under Federal Rules Evidence 403, a judge may exclude relevant evidence if he finds that the probative value of the evidence "is substantially outweighed by the danger of unfair prejudice, confusion of the issues, misleading the jury, or by considerations of undue delay, waste of time or needless presentation of cumulative evidence." Here, it is quite likely that the probative value of the psychiatrist's described testimony is substantially outweighed by the danger of unfair prejudice.

Defense asks plaintiff's doctor of past bodily condition relating to a prior accident. Admissible?

No, because the question asks the doctor to relate a statement of a past bodily condition. Discussion of correct answer:Under Federal Rule of Evidence 803(4), a statement made "for the purposes of medical diagnosis or treatment and describing medical history, or past or present symptoms, pain or sensation" is admissible into evidence. Thus, the judge will not sustain the objection, because the question is an attempt to elicit a statement of a past bodily condition. Incorrect. Yes, because requiring the doctor to answer the question would violate the doctor-patient privilege. The teacher waived the doctor-patient privilege with regard to her shoulder injury when she put her physical condition at issue by bringing a claim of personal injury. A plaintiff may not put her physical condition at issue in a lawsuit and then refuse to disclose relevant evidence relating to her physical condition by asserting the doctor-patient privilege.

Insurance Company asks lawyer to ask engineer to prepare report for lawyer. Can plaintiff compel production of documents?

No, because the report is protected under the attorney-client privilege. Discussion of correct answer:Under the attorney-client privilege, any person who engages in a confidential communication with an attorney (or someone who the person reasonably believes is an attorney) for the purpose of seeking legal advice or representation (regardless of whether the attorney agrees to accept the request for advice or representation) has a privilege that allows that person to prevent anyone from testifying about the confidential communication. The attorney-client privilege exists until it is waived, and the privilege even extends beyond the life of the client. While the client is the holder of the privilege, absent waiver by the client, the attorney must assert the privilege to protect the client. If the client is a corporation, the privilege extends to confidential communications by officers or employees of the corporation if they were authorized or requested by the corporation to make the communication. Here, the train company's supervisor prepared the report at the request of the train company's attorney. Accordingly, the report would fall within the attorney-client privilege and is therefore protected from compelled production.

Husband and Wife divorced, can wife testify against Husband for things said while married?

No, because the testimony would violate the marital communication privilege. Discussion of correct answer:The marital communication privilege allows either spouse to refuse to disclose or prevent the other spouse from disclosing confidential communications made between the spouses during their marriage. Unlike the spousal privilege that applies in criminal cases, the marital communication privilege does not terminate when the marriage ends. Even after the spouses divorce, the marital communication privilege still protects from disclosure any confidential communications between the spouses made during the marriage. Here, any statement the wife made to the husband regarding the dog's likelihood of biting others was made during their marriage. Therefore, the husband can assert the marital communication privilege to prevent the wife from answering the plaintiff's lawyer's question. The marital communication privilege protects from disclosure confidential communications made between spouses during the marriage. Either spouse may refuse to disclose or may prevent the other spouse from disclosing those communications. The privilege survives a divorce. Here, any statements the wife made to her husband during the marriage about the dog's likelihood of biting someone were protected by the privilege. The divorce does not remove that privilege.

Student's lawyer attempted to rehabilitate witness with questions regarding truthfulness character. But witness had only been impeached for bias earlier. Admissible?

No, because the witness's credibility for truthfulness was not attacked. Discussion of correct answer:Opinion or reputation evidence regarding a witness' character for truthfulness may be admitted when the witness' credibility for truthfulness has been attacked. In this case, the dentist's lawyer didn't impeach the student's witness on grounds of the witness' credibility for truthfulness but rather with evidence of potential bias. Therefore, evidence of the witness' character for truthfulness cannot be admitted and this answer is correct. Incorrect. Yes, because opinion or reputation evidence can be used to rehabilitate an impeached witness. This answer is tempting but incorrect. Reputation or opinion evidence concerning a witness' character for truthfulness can be used to rehabilitate an impeached witness. But only if the evidence used to impeach the witness attacks that witness' credibility for truthfulness. If the impeachment evidence attacks the witness' credibility on other grounds, such as bias, then evidence regarding character for truthfulness is not admissible. In this case, the witness' credibility was attacked with evidence of potential bias, and, therefore, the reputation or opinion evidence of character for truthfulness is not admissible. Thus, this answer is not correct.

Grand Jury Testimony of a dead man, admissible?

No, because the witness's grand jury testimony is hearsay not within an exception. Discussion of correct answer:Prior grand jury testimony is does not fall within the exception for former testimony because the party against whom the testimony is offered (the defendant here) did not have a full opportunity to and similar motive in examining the witness before the grand jury. During grand jury proceedings, the defendant is not afforded any opportunity to examine witnesses or otherwise put evidence before that body. The grand jury proceedings are controlled exclusively by the prosecutor without any participation by the defendant.

Def's friend got drugs, sold to another guy and fled. Can the other guy say that the friend got the drugs from the Def?

No, because there is no corroboration of the statement. Discussion of correct answer:Under FRE 801, hearsay is a statement that: (1) the declarant does not make while testifying at the current trial or hearing; and (2) a party offers in evidence to prove the truth of the matter asserted in the statement. Under FRE 804(b)(3), there is a hearsay exception for statements against interest if the declarant is unavailable as a witness. A statement against interest is one that a reasonable person in the declarant's position would have made only if the person believed it to be true because it was so contrary to the declarant's proprietary or pecuniary interest or had so great a tendency to invalidate the declarant's claim against someone else or to expose the declarant to civil or criminal liability. If offered in a criminal case, it must be supported by corroborating circumstances that clearly indicate its trustworthiness if it tends to expose the declarant to criminal liability. Here, the friend's statement is hearsay, and the prosecutor wants it admitted to prove that the defendant made at least one drug sale. The friend is unavailable to testify. The friend's statement was made against interest because it exposes him to his own criminal liability for possession of illegal narcotics. In a civil case, it would be admissible as a hearsay exception without more. However, because this is a criminal case, it cannot be admitted as a hearsay exception because there is no corroborating circumstance to clearly indicate its trustworthiness. The court will not allow the witness to recite the friend's statement. nder FRE 804(b)(3), there is an exception for statements against interest if the declarant is unavailable as a witness. However, if offered in a criminal case, it must be supported by corroborating circumstances that clearly indicate its trustworthiness if it tends to expose the declarant to criminal liability. Here, the prosecutor has no corroborating evidence to support the statement that the friend bought some illegal narcotics from the defendant. Thus, a hearsay exception exists, but its requirements are not met.

An offer on the scene to pay medical expenses -admissible?

No, if it is offered only to show the driver's liability for the accident. Discussion of correct answer:Federal Rule of Evidence 409 allows evidence of a statement or promise to pay medical costs to be admitted for the purpose of showing ownership or control, but not for the purpose of proving liability on the part of the person making the offer. This is because public policy does not favor discouraging people from making offers of settlement or payment by allowing the offers to be used against them later. In this case, the driver offered only to pay any medical costs the student incurred. As such, the student's testimony about the driver's promise cannot be admitted. Federal Rule of Evidence 801(d)(2) allows statements of a party opponent to be introduced into evidence. However, the driver's offer to pay the student's medical expenses, without more, would not be considered to be a statement of his liability for the accident

Wife charged with manslaughter, husband filed for divorce soon after. At trial, husband asserts spousal privilege and refuses to testify. Can he?

No, the husband and wife are not married. Discussion of correct answer:There are two spousal privileges: (1) the spousal testimony privilege; and 2) the spousal communications privilege. Neither privilege applies here. The spousal testimony privilege prevents a person from being forced to testify in a criminal prosecution against their spouse. The privilege is asserted by the testifying spouse, rather than the accused. The privilege encompasses all subject matters of testimony, but it endures only as long as the marriage. Upon divorce or annulment, the privilege no longer applies even to events that occurred during the marriage. Unlike the spousal testimony privilege, the spousal communication privilege continues to apply after divorce. Either spouse may refuse to testify or bar the other from testifying as to communications made between them during the marriage. This privilege, however, is not applicable to the facts described here because the husband will be testifying as to what he saw the wife do rather than any communication. Incorrect. Yes, the husband and wife were married at the time of the death. Whether the husband and wife were married at the time of the death is important in analyzing whether a spousal communications privilege would apply. It, however, is irrelevant in determining whether the spousal testimony privilege would apply. The relevant inquiry for that privilege is only whether or not the husband and wife are married at the time of the trial.

A statute enacted by the state concludes that a person with the specified blood alcohol level of 0.05% or higher is under the influence. However, such a presumption may not be constitutionally instated and forced onto the jury. Can it?

No, the jury may find the driver not guilty, even if he introduces no evidence on the issue of "under the influence." Discussion of correct answer:A presumption is a proposition that the jury is required to accept, unless properly rebutted by evidence to the contrary. Some presumptions are conclusive such that no contrary evidence will be allowed to rebut the presumption. Conclusive presumptions are rare and usually exist because of strong public policy considerations. Conclusive presumptions are not permitted in criminal cases as to any prima facie element of the crime; the prosecution must prove every element of the crime beyond a reasonable doubt and the defense must be permitted to offer evidence to establish reasonable doubt as to any such element of the crime. Here, the statute enacted by the state concludes that a person with the specified blood alcohol level of 0.05% or higher is under the influence. The showing of the blood test (the foundational fact) appears to require the jury to find that the driver was under the influence of alcohol when he injured the pedestrian. However, as noted above, such a presumption may not constitutionally be applied against a criminal defendant. Thus, even though the prosecution established the foundational facts, the jury cannot be required to find that the driver was under the influence of alcohol, even if he produces no evidence on the subject. Therefore, this answer is correct. While a conclusive presumption ordinarily controls the jury's findings once the foundational facts are shown, constitutional limitations preclude the application of a conclusive presumption to establish an element of a charged crime against a criminal defendant. Thus, the jury cannot be compelled to make a finding that the driver was under the influence of alcohol by proof of his blood alcohol level. The jury may so find based on the evidence, but the jury may also find that the driver was not under the influence. Whether the driver produces evidence on the subject is also immaterial. Even if the driver presents no defense, the prosecution will be required to prove every element of the charged crime beyond a reasonable doubt. Therefore, the jury is not required by the presumption to find the driver guilty and this answer is incorrect.

Testimony from roommate of victim that she heard on recording that she was heading out with Defendant to dinner at 15 before crime. Admissible?

No, unless it is established that the tape was accidentally destroyed or inadvertently lost. Discussion of correct answer:Under FRE 1002, in order to prove the contents of a writing, recording, or photograph, the original writing, recording, or photograph is required, subject to numerous exceptions. Under FRE 1004, the original is not required, and other evidence of its contents is admissible, if the original writing, recording, or photograph is unavailable because: (1) it has been lost or destroyed (absent loss or destruction by the proponent acting in bad faith); (2) it cannot be obtained by any available judicial process or procedure; or (3) it was in the possession of the party against whom offered, and who was notified (by the pleadings or otherwise) at the time of his possession that the original would be required at the hearing, and he did not produce it. The problem is that in using the roommate to introduce this evidence, the prosecution violated FRE 1002 that requires the original, unless the original is lost or destroyed as described in FRE 1004. Incorrect. Yes, because the testimony may contradict the man's alibi defense. There is no evidentiary rule that allows identity evidence or contradictions of alibi evidence in any form. The problem is that in using the roommate to introduce this evidence, the prosecution violated FRE 1002 that requires introduction of the original recording, unless the original has been lost or destroyed as described in FRE 1004.

Past Recollections Recorded vs Simple Refresh

Overrule the objection, because the invoices were only used to refresh the CFO's recollection. Discussion of correct answer:The Federal Rules of Evidence provide that on direct examination, the examiner may, upon laying a proper foundation, jog the memory of a witness who has some knowledge of the purported testimony but whose memory is incomplete. Any writing, photograph or other form of evidence which refreshes the witness's recollection will suffice. After the memory has been "refreshed," the witness must testify without having the refreshing evidence present. In addition, the document used to refresh a witness's memory must be made available to the opposing party. In this case, the CFO would be in a position to know how many units the manufacturer shipped but could not recall the specific number. It is permissible for the CFO to use the invoices to aid her recollection. Therefore, the testimony is admissible and the manufacturer's objection should be overruled. The Federal Rules of Evidence provide that a memorandum or record concerning a matter about which a witness once had knowledge but now has insufficient recollection to enable the witness to testify fully and accurately may be admitted as evidence, if shown to have been made or adopted by the witness when the matter was fresh in the witness's memory. This rule is an exception to the hearsay rule which bars admission of out-of-court oral or written statements offered to prove the truth of the matter asserted. If admitted, the memorandum or record may be read into evidence but may not itself be received as an exhibit unless offered by an adverse party. In this case, the CFO only used the invoices to refresh the CFO's memory as to the specific number of units. The CFO was otherwise able to testify. Therefore, it would not be necessary to admit the invoices as a recorded recollection under the federal rules of evidence.

Police officer reporting on interaction between informant and defendant he witnessed. Can he say what the informant said to arrange the drug deal? Objection?

Overruled, because the evidence is not being offered for its truth. Discussion of correct answer:Hearsay is an out-of-court statement offered to prove the truth of the matter asserted. Where a statement is being offered for a non-truth purpose, it is not hearsay. Verbal acts are words that have legal significance apart from their truth; verbal acts are not hearsay. The police officer testified that the confidential informant ordered illegal drugs from the defendant. The purpose of the police officer's testimony was to explain how the controlled drug buy occurred, and the evidence was not being offered to prove the truth of the confidential informant's statement of ordering the drugs. Therefore, the testimony is not hearsay because it is not being offered for its truth, and the court should overrule defense counsel's hearsay objection. A witness may only testify to a matter if the witness has personal knowledge of the matter. It is accurate that the police officer has personal knowledge of the conversation between the confidential informant and the defendant, and the police officer was therefore qualified as a witness to testify to the matter. However, the defense counsel's objection to the testimony was a hearsay objection. Therefore, the better answer choice is the one that specifically explains why the hearsay objection fails.

Defense calls coworker to testify at what time eyewitness said to coworker about defendant's claim to have been at home on night of murder." Can objection to be sustained? Eyewitness wasn't cross examined by def when this exact thing came up.

Sustained, because defense counsel did not cross-examine the eyewitness. Discussion of correct answer:A witness may be impeached with a prior inconsistent statement. However, extrinsic evidence of the witness's prior inconsistent statement is only admissible if the witness has an opportunity to explain or deny the prior statement, and the adverse party has an opportunity to examine the witness about it. The eyewitness testified that the eyewitness got home from work at 6:00 p.m., and the defense did not cross-examine the eyewitness about the prior statement to the co-worker. The eyewitness did not have an opportunity to explain or deny the statement to the co-worker, and the prosecution did not have an opportunity to examine the eyewitness about the statement. Therefore, extrinsic evidence of the eyewitness's prior inconsistent statement, such as the co-worker's testimony, is inadmissible to impeach the eyewitness. Incorrect. Overruled, because the defense may impeach the eyewitness with the eyewitness's prior inconsistent statement. This answer choice is incorrect because the requirements for admitting extrinsic evidence of a witness's prior inconsistent statement have not been met. For extrinsic evidence of a witness's prior inconsistent statement to be admissible to impeach the witness, the witness must be given an opportunity to explain or deny the prior statement, and the adverse party must be given an opportunity to examine the witness about the prior statement. Because the defense did not cross-examine the eyewitness, the eyewitness did not have an opportunity to explain or deny the prior statement to the co-worker, and the defense cannot present extrinsic evidence (i.e. the co-worker's testimony) of the eyewitness's prior inconsistent statement.

Agent offering opinion on mental state of defendant. Objection held?

Sustained, because it is improper to prove intent by opinion testimony. Discussion of correct answer:According to FRE 704, "No expert witness testifying with respect to the mental state or condition of a defendant in a criminal case may state an opinion or inference as to whether the defendant did or did not have the mental state or condition constituting an element of the crime charged or a defense thereto." Determination of such issues is a matter for the trier of fact alone. Therefore, the agent may not render an expert opinion as to the defendant's intent to distribute narcotics.

Defense lawyer offers map showing shortest root between client's alibi location and scene of crime on night of the stabbing. Results seem reputable. How should the court go on Judicial Notice?

That it must accept the distance as conclusive. Discussion of correct answer:In this case, the court would take judicial notice of the distance between the two places because a court must take judicial notice if a party requests it and the party supplies appropriate information. The court would then direct the jury to accept the distance as conclusive. Federal Rule of Evidence 201 states that, in a civil case, the court shall instruct the jury to accept as conclusive any fact judicially noticed. Incorrect. That it may, but does not have to, accept the distance as conclusive. This answer is not correct because: 1) The court should take judicial notice of the distance between the two places because a court must take judicial notice if a party requests it and the party supplies appropriate information; and 2) Facts judicially noticed are deemed to be proved conclusively in a civil case. Therefore, the jury does not have a choice of rejecting the information about the distance between the bowling alley and the bar.

Which question may the defense ask the farmhand receiving a lesser sentence for testifying against farmer defendant?

The correct answer is: Have you ever been convicted of a felony? Discussion of correct answer:Under the Federal Rules of Evidence, evidence of prior felony convictions occurring less than 10 years ago can be used to impeach a witness's credibility. As such, this question is proper. Incorrect. Have you ever been convicted of perjury or disorderly conduct (a misdemeanor)? Although it would be proper to ask the farmhand whether he had been convicted of perjury, a question regarding a misdemeanor conviction not involving dishonesty would not be allowable. As disorderly conduct is a misdemeanor and not an offense that reflects on a person's honesty, this question would not be permissible.

Plaintiff says Def hit him. Defendant seek sot introduce testimony by witness who saw plaintiff in his car say on his phone "dropped my phone and I then hit that car picking it up. Admissible."

The correct answer is: Yes, for impeachment purposes and to prove that the plaintiff contributed to the accident. Discussion of correct answer:Under the Federal Rules of Evidence, certain types of out-of-court statements are classified as nonhearsay, even when offered for their truth. Out-of-court statements that are nonhearsay include: (1) statements by an opposing party; (2) prior sworn inconsistent statements; (3) prior consistent statements; and (4) prior identifications. In this case, the witness's testimony involves the plaintiff's own statement, and which is now being offered against the plaintiff at trial. Therefore, the statement constitutes a statement by an opposing party, which is admissible nonhearsay. As such, the statement is fully admissible, for all appropriate purposes, both to impeach the plaintiff and/or as substantive evidence. Incorrect. Yes, for impeachment purposes only. Under certain circumstances, an out-of-court statement may be admissible only for a limited purpose, such as to impeach a party or witness. However, the Federal Rules of Evidence classify certain types of out-of-court statements as nonhearsay, even though they are offered for their truth. In this case, the witness's testimony involves the plaintiff's own statement, which is now being offered against the plaintiff at trial. As such, the statement constitutes a statement by an opposing party, which is fully admissible nonhearsay. As non-hearsay, the statement is admissible not only for the limited purpose of impeaching the plaintiff, but for any permissible purpose.

The neighbor is brought in to testify her husband told her it was light out that night. Admissible?

The correct answer is:Deny the motion, regardless of whether the neighbor's husband is available to testify. Discussion of correct answer:Pursuant to Federal Rule of Evidence 803(1), a statement made by the declarant to describe or explain a condition or event and made while the declarant is perceiving the event or shortly thereafter, is admissible as an exception to the rule excluding hearsay evidence. This is known as the "present sense impression" exception to the hearsay rule. In this case, very little time elapsed between the time that the neighbor's husband (the declarant) observed the moon and the time that he expressed his perception to his wife. As such, the neighbor's testimony is admissible under the present sense impression to the hearsay rule, and the motion to strike should be denied. Incorrect. Grant the motion, because the neighbor did not have firsthand knowledge of the matter. While the neighbor herself may not have observed the moon on the night in question, she does have personal knowledge of what her husband said to her regarding the moon. As an out-of-court statement by a declarant (the neighbor's husband), offered to prove the truth of the matter asserted (that is, that the moon was shining brightly on the night in question), the neighbor's testimony is hearsay. Thus, if the testimony is inadmissible, it will not be for the neighbor's lack of personal knowledge but because the testimony is inadmissible hearsay evidence. This answer additionally neglects to address whether the testimony falls within any of the exceptions to the rule excluding hearsay evidence.

12 years before the woman had been convicted by a similar crime to which she is on trial. How may this be introduced?

The correct answer is:Evidence of the woman's prior criminal conviction is admissible to attack the woman's character for truthfulness if the probative value of the evidence is greater than its prejudicial effect. Discussion of correct answer:In a criminal case in which the witness is a defendant, evidence of a prior criminal conviction for a crime punishable by death or imprisonment for more than one year is admissible to attack the witness's character for truthfulness if the probative value of the evidence outweighs its prejudicial effect. The test for admissibility here is whether the probative value is greater than the prejudicial effect. This is a criminal case, and the woman (the defendant) testified as a witness in her own defense. The woman's prior conviction was for a crime punishable by more than one year because the facts state that the woman served three years in prison. The 10-year rule does not apply because the woman was released from confinement nine years ago. Thus, the prosecution could impeach the woman by attacking her character for truthfulness with evidence of the prior conviction if the probative value of the evidence is greater than its prejudicial effect. Incorrect. Evidence of the woman's prior criminal conviction is admissible to attack the woman's character for truthfulness if the probative value of the evidence is not substantially less than the risk of unfair prejudice. This answer choice is incorrect because it does not apply the correct test for admissibility of the evidence of the prior conviction for impeachment purposes. In a civil case or a criminal case in which the witness is not the defendant, a witness's character for truthfulness may be attacked by evidence of a prior conviction for a crime punishable by death or imprisonment for more than one year if the evidence passes the Rule 403 balancing test. The Rule 403 balancing test provides that relevant evidence may be excluded if its probative value is substantially outweighed by the danger of, among other things, unfair prejudice. In other words, under the 403 balancing test, evidence is admissible if the probative value is not substantially less than the risk of unfair prejudice. This is a criminal case, and the woman (the defendant) is the witness. Thus, this is not the applicable test in these circumstances.

Secretary said she dropped off the package at the last minute after plaintiff expressly told her of it and the deadline. May she bring in this statement by the plaintiff in?

The correct answer is:No, because the tenant's statement to the secretary is hearsay not within any exception. Discussion of correct answer:Under FRE 801(c), hearsay is: (1) any statement other than a statement made by the declarant while testifying at the trial or hearing in which the statement is proffered; (2) which is offered into evidence in order to prove the truth of the matter asserted. Here, the question tells us that the statement is offered to prove the truth of the matter asserted, i.e., that the packet contained the notice exercising the option to purchase the building. Incorrect. Yes, because the tenant's statement to the secretary is non-hearsay. Here, the question tells us that the statement is offered to prove the truth of the matter asserted, i.e., that the packet contained the notice exercising the option to purchase the building. Thus, under FRE 801(c), this is hearsay.

Roommate to testify about the unsanitary practices of the cook in the kitchen. Admissible for this food poisoning case? Objection sustained?

The correct answer is:Overruled, because the cook was employed at the restaurant, and his duties included keeping the kitchen clean. Discussion of correct answer:Hearsay is an out-of-court statement offered to prove the truth of the matter asserted. However, a statement is not hearsay when the statement is offered against an opposing party, and the statement was made by the opposing party's employee on a matter within the scope of employment. The cook was an employee of the restaurant, and during his employment, he made statements to the roommate regarding his unsanitary practices in the kitchen. The statements were within the scope of the cook's employment because his duties included keeping the kitchen clean. Therefore, the statements are not hearsay. Incorrect. Overruled, because the cook's statements are against his interest. There is a hearsay exception that applies where the declarant is unavailable as a witness, and the statement is against the declarant's interest such that a reasonable person in the declarant's position would not have made the statement unless it were true. While the cook's statement may be against his interest, there is nothing in the facts to indicate that the cook is unavailable as a witness. Further, because the statement is offered against the restaurant, and the statement was made by the cook concerning the scope of his employment and while he was employed with the restaurant, the cook's statement is not hearsay. Because the statement is not hearsay, there is no need to consider whether a hearsay exception applies.

Def on trial for robbery, and his ex-wife was brought in to testify about what he told after the robbery which was while they were married. Can def stop on basis of marital privilege and spousal privilege?

The correct answer is:Sustain the objection as to the marital privilege, but overrule the objection as to the spousal privilege. Discussion of correct answer:The distinction between the spousal and marital privileges is often tested on the MBE. The spousal privilege only applies in criminal cases when the testifying spouse is married to the defendant spouse at the time of trial. Here, the ex-wife is not married to the defendant at the time of trial, so the spousal privilege does not apply. However, the statements made to the wife on the day of the robbery occurred during the marriage. The marital privilege covers all conversations occurring during the marriage, and survives a divorce. In addition, either spouse or ex-spouse may claim the privilege, which the defendant does here. Incorrect. Sustain the objection as to both privileges. This choice is incorrect because the spousal privilege only applies in criminal cases when the testifying spouse is married to the defendant spouse at the time of trial. Here, the ex-wife is not married to the defendant at the time of trial, so the spousal privilege does not apply.

Defendant wishes to say that he told the police officer that night when asked that he had been out fishing with his grandfather. Should court overrule prosecution objection to that?

The correct answer is:Sustained, because the defendant's testimony is hearsay not within any exception. Discussion of correct answer:Under Federal Rule of Evidence 801(d)(1)(B), a declarant's prior statement that is consistent with his testimony is not hearsay if the declarant testifies at the trial or hearing and is subject to cross-examination concerning the statement, and the statement is offered to rebut an express or implied charge against the declarant of recent fabrication or improper influence or motive, or one of identification if offered to rebut a charge of recent fabrication or improper influence or motive. If these conditions are met, the statement may be used as substantive evidence. Unlike with prior inconsistent statements, it is not required that the prior consistent statement have been made while the declarant was under oath. Here, the defendant is attempting to testify to an out-of-court statement offered to prove the truth of the matter asserted therein (that is, that he was on a fishing trip with his grandfather at the time that the rapes occurred). Given that the defendant is testifying during his case-in-chief, the prosecution cannot have yet introduced impeaching evidence for the defendant to rebut. Thus, it is clear that the defendant is not offering this prior consistent statement to rebut a charge of recent fabrication or improper motive. As such, his testimony is inadmissible hearsay, and the prosecution's objection should be sustained. Incorrect. Overruled, because the defendant's answer would constitute a prior consistent statement. Prior consistent statements may not be introduced merely to bolster the credibility of a witness. Pursuant to Federal Rule of Evidence 801(d)(1)(B), a defendant may not support his own credibility by introduction of a prior consistent statement unless this statement is offered to rebut a charge of recent fabrication or improper influence or motive.

Employee took picture of crash pile-up. Person involved in crash furious and sues. Employee is MIA. Who can authenticate the picture best?

The correct answer is:Testimony from another skier who disembarked from the ski lift immediately before the plaintiff did, who observed the entire incident. Discussion of correct answer:As a general rule, a document is not admissible unless it has been authenticated (i.e., sufficient proof offered that the document is what it is purported to be). To establish that a document is genuine, Federal Rule of Evidence 901 provides that a party may authenticate the document by admission or stipulation of the opponent, by use of a witness, by jury comparison, or by circumstantial evidence. The testimony of another skier who disembarked from the chair lift immediately before the skier and observed the entire incident, would be the most helpful in verifying that the scene depicted in the photograph is indeed the scene of the accident in question. The Federal Rules do not require that the photographer himself authenticate the photograph. Incorrect. Testimony from the person who developed the pictures. The person who developed the pictures would be able to testify to the "chain of custody" for the photograph. However, this person would not be able to identify whether the photograph actually depicts the location of the skier's mishap. Thus, this answer is incorrect.

Defendant telephoned Plaintiff and explained his windshield wipers weren't working due to crash, and offered a settlement. Which can be admitted?

The correct answer is:The defendant's statement about being blinded by the hailstorm and his statement concerning payment of hospital expenses are both inadmissible as an offer to compromise. Discussion of correct answer:One of the key testing areas in Evidence is Rule 408, which deals with settlement or compromise negotiations. Always remember that in order to exclude statements or admissions made during settlement negotiations, there must be an actual dispute between the parties. In this question, there is an actionable dispute between the parties. The facts indicate that the defendant's car struck the woman as she was crossing an intersection. The woman was injured in the accident and subsequently hospitalized. Thereafter, the defendant learned that the woman had retained an attorney. The defendant then made a settlement offer in an attempt to forestall litigation. Under Rule 408, any statements made during the course of settlement negotiations will be excluded. Thus, choice (C) is correct. It is important for students to distinguish between Rule 408 and Rule 409. Under Rule 409, statements made in connection with an offer to pay medical expenses are severed and admitted. Choice (B) is incorrect because even though the defendant is offering to pay the woman's medical expenses, the offer is really being made in an effort to settle their dispute.

Ethtician spoke with witnesses before being married. May she be compelled to testify?

The correct answer is:Yes, as a statement by the esthetician. Discussion of correct answer:Under Federal Rule of Evidence 801(d)(1)(c), prior statements of a party, offered against the party, are nonhearsay and are admissible. Here, given that the motorcyclist named the esthetician as one of the parties to his suit, her out-of-court statement is admissible against her as a statement of an opposing party. Incorrect. Yes, because the testimony will probably impeach the dog groomer's testimony. Given the facts presented, the esthetician's testimony would be more likely to corroborate rather than impeach the dog groomer's testimony. As such, the testimony would not be admissible to impeach the dog groomer's testimony.

State Charges on Robbery he was acquitted, but at Bar he celebrated with friends and blurted out his part in the robbery. Federal court seeks to use this in Federal Court. Can they?

The correct answer is:Yes, as an opposing party's statement. Discussion of correct answer:Students must be aware that the bar examiners have a preference for the nonhearsay answer choice over the exception-to-the-hearsay rule answer choice. If the statement constitutes nonhearsay, then it is obviated from needing a hearsay exception for admission. In other words, in analyzing the bases for a statement's admission that appears to have a hearsay problem, first see if it constitutes nonhearsay. If not, and the statement is hearsay, then look to see if a relevant exception applies. Here, the defendant's statement is clearly against his penal interest under Rule 804, and the defendant is unavailable to testify as required by Rule 804 by virtue of his present ability to invoke his Fifth Amendment right against self-incrimination. However, his statement is also nonhearsay--it is a statement by a party offered by the opponent party, and the Federal Rules of Evidence define opposing party's statements as nonhearsay. The bar examiners prefer (D) over (C) because (D) is the nonhearsay answer. Incorrect. Yes, as a statement against interest. This is not the best answer because, where both apply, preference is given to the nonhearsay answer over the exception-to-the-hearsay rule answer.

Former airline employee testifying that no ticket was sold to customer based on website. Admissible?

The correct answer is:Yes, as direct evidence of a matter in issue. Discussion of correct answer:One of the Rule 803 hearsay exceptions is that the absence of an entry in records, reports, or data compilations can be used to prove the non-existence or non-occurrence of a matter, if the matter was of the kind normally kept in those records, reports, or compilations. An airline would necessarily have a system of recording who had bought a ticket, and the absence of a person's name in that system would be admissible as a hearsay exception. Incorrect. No, because the representative is a former employee. The representative's former-employee status would affect admissibility only if it involved a hearsay declaration that was asserted as a statement by the airline company and was made when the representative was not employed. However, the representative's testimony is not hearsay because the statement is being made in court. The non-existent ticket record is subject to a hearsay exception. Therefore, this element of the rules applicable to statements is immaterial. Thus, this answer is incorrect.

Man is cheating on later victim (a hardened convict) with victims wife. Wife had told man about how the victim was in prison for stabbing another guy she was with. Later calls Man to say victim is loose and know. Man later kills victim. Is wife's word on victim stabbing last guy admissible to prove effect on lover at the time?

The correct answer is:Yes, as nonhearsay. Discussion of correct answer:While evidence offered for one purpose may be hearsay, for another purpose it may be permissible. Here, the statement is not offered to prove the truth of the matter--i.e., that the husband tried to stab someone (which would be inadmissible hearsay). Instead, the statement is offered to show its effect on the lover's state of mind. That is, because the husband was so jealous in the past that he tried to stab someone he suspected of being his wife's lover, he would try to kill her current lover, too. Consequently, the statement is relevant to prove the self-defense plea. It is admissible nonhearsay. Incorrect. No, as hearsay not within any exception. The statement is not hearsay evidence; hearsay is an out-of-court statement offered to prove the truth of the matter asserted. The woman's statement about her husband is not being offered to prove that the husband tried to stab the coworker. It is being offered to show the lover's state of mind when the husband appeared at the restaurant, and is therefore relevant to prove the self-defense plea.

May defendant's girlfriend refrain being forced to testify as to his clothing on morning of the crime if they married after the crime?

The correct answer is:Yes, because she has the right to refuse to testify. Discussion of correct answer:In answering this evidence question, it is necessary to differentiate between the spousal communication privilege, choice (A), and the spousal testimony privilege, choice (B). The spousal communication privilege protects confidential communications between the spouses made during the marriage. The spousal testimony privilege, on the other hand, prevents one spouse from testifying against the other in a criminal case. The spousal testimony privilege or incapacity blocks adverse testimony based on knowledge gained in any way, at any time, as long as the accused and the proposed witness are married at the time of trial. As such, the spousal testimony privilege applies to "pre-marital" acts. Choice (B) is correct because the relevant issue regarding the man's clothing occurred before the marriage. Also, with respect to the spousal testimony privilege, it is the witness-spouse who is the holder of the privilege, not the accused.

Can the physician testify about the communications made during the examination?

The correct answer is:Yes, because the cyclist put his medical condition at issue by filing suit. Discussion of correct answer:The physician-patient privilege allows a patient to refuse to disclose and prevent any other person from disclosing confidential communications made for the purposes of diagnosis or treatment. The physician-patient privilege generally yields when a patient initiates litigation concerning his medical condition, and is most commonly waived when a plaintiff voluntarily places his medical condition in issue by filing a complaint alleging that he suffered physical injuries. It is clear from the context of this question that the cyclist has initiated a lawsuit which puts his medical condition at issue. It would be impossible for a defendant to defend against a personal injury lawsuit if all evidence related to the alleged injuries were unavailable. Thus, the physician can testify and this answer is correct. Incorrect. No, because the physician-patient privilege applies. The physician-patient privilege does protect confidential communications between a patient and physician for the purpose of diagnosis and treatment, but not when the plaintiff has put his medical condition and personal injuries at issue in a litigation. Because the cyclist has put his physical condition at issue here, the privilege would not apply, and this answer is not correct.

Trash collector got into a scuffle while on duty. Plaintiff sought to bring in testimony from an organization that had moved against this trash collector. Activist from org would testify about time collector hit a demonstrator. Inadmissible?

The correct answer is:Yes, because the witness's testimony constitutes inadmissible character evidence. Discussion of correct answer:Under Federal Rule of Evidence 404(a), evidence of a person's character "is not admissible for the purpose of proving that he acted in conformity therewith on a particular occasion." The facts here do not show that this testimony would be offered for any other reason (such as the MIMIC reasons of motive, intent, absence of mistake, identity, or common plan or scheme). As such, the evidence constitutes inadmissible character evidence. Incorrect. No, because the witness's testimony is relevant to a material issue in the case. The trash collector's propensity for violence is not a material issue in the case. In a civil action for damages arising from a physical attack, the defendant's character for violence would be relevant only to show that he acted in conformity with that character on the occasion in question, which is a prohibited use of character evidence.

Plaintiff, suing over new assignments against his contracted specifications, wishes to provide court wish map showing how much further his new assigned sale turf is. Should the court take judicial notice of this?

The court must take judicial notice of the distance between the cities, because the salesman's attorney made a proper request and furnished the court with an accurate map. Discussion of correct answer:Judicial notice is a substitute for evidence where the court accepts certain matters as true without the need for formal proof. A judge has discretionary use of judicial notice when no request is made. However, if a party requests that the court take judicial notice, the court must do so if the party provides sufficient information to the court that enables the court to take notice. In this case, the salesman's attorney did request that the court take judicial notice and provided an accurate map. Therefore, judicial notice is mandatory and this answer is correct. The court may, but is not required to, take judicial notice of the distance between the cities. Federal Rules of Evidence establish the conditions under which judicial notice may or must be taken. A judge has discretion to take judicial notice even when no party makes a request for judicial notice. However, if a party does request that the court take judicial notice of a matter, the court must do so if the party provides sufficient information to the court that enables the court to take notice. Judicial notice then becomes mandatory. In this case, the salesman's attorney did request judicial notice of the distance between the cities and provided an accurate map. Therefore, judicial notice is mandatory and not discretionary in this case, and this answer is incorrect.

At trial, a witness was asked "isn't it true that the traffic light for northbound traffic was green when the collision occurred?"

The defendant's attorney, during the cross-examination of the defendant who had been adversely called by the plaintiff. Discussion of correct answer:Rule 611(c) of the Federal Rules of Evidence states: "Leading questions should not be used on the direct examination of a witness except as may be necessary to develop the witness' testimony. Ordinarily leading questions should be permitted on cross-examination. When a party calls a hostile witness, an adverse party, or a witness identified with an adverse party, interrogation may be by leading questions." Note that the entire matter of whether or not to allow leading questions is discretionary, and the judge's action will not be reviewed unless it is charged that it contributed to the denial of a fair trial. Choices (A) and (C) present situations where an adverse party (or the spouse of an adverse party) is being asked a leading question by opposing counsel. In this situation, a hostile exchange can be predicted. Consequently, the general rule is that the sponsoring counsel can, from the outset, treat the adverse party as a hostile witness. In each of these cases, an objection to the leading question will be overruled. Likewise, choice (B) presents the ordinary situation of a leading question being asked of a witness on cross-examination. Here, an objection to the leading question will also be overruled, because it is assumed that the cross-examiner and the witness are antagonistic. Choice (D), however, presents a different situation. In this example, the defendant has already been called by the plaintiff as an adverse witness. As discussed, leading questions would be proper on direct examination. However, now on cross-examination, the defendant is being questioned by his own attorney, and there is a danger that the defendant would appear to be biased in favor of the cross-examiner. There is no right for the party to be further examined immediately by his own counsel, but the judge has discretion to permit it or to require that his examination be deferred until the witness-party's own case is put on. Choice (D) is therefore the correct answer, because the objection will be sustained.

Defendant arrested for burglary, when arrested there was cocaine in his backpack. May Prosecution bring in?

The evidence should be excluded, because the probative value is substantially outweighed by the danger of unfair prejudice. Discussion of correct answer:Under Federal Rule of Evidence 404, past crimes or other wrongful acts of a defendant are admissible as circumstantial proof of an element of the charged crime, such as motive, intent, absence of mistake, identity, or common scheme or plan (the "MIMIC" rule). Pursuant to Federal Rule of Evidence 403, evidence may be excluded if its probative value is substantially outweighed by the danger of unfair prejudice. In this question, possession of cocaine does not appear to have any connection with burglary. Thus, evidence of the man's possession of cocaine is inadmissible, because it would be more prejudicial than probative. Under Federal Rule of Evidence 404, character evidence may not be admitted for the purpose of showing that the defendant is of a bad character and, as such, has a propensity to commit crimes. The prosecution is permitted, in a very limited set of circumstances, to use character evidence circumstantially (i.e., to infer that on a specific occasion, the defendant acted in conformity with the character trait), as where the defendant has first introduced evidence of his good character as being inconsistent with the charged crime. The prosecution may also introduce evidence of the defendant's other crimes as circumstantial evidence of motive, intent, absence of mistake, identity, or common scheme or plan. However, showing the man's propensity to commit crimes is not a permissible purpose for admitting the evidence of his possession of cocaine.

Plaintiff charged with knife-point rape of a woman. Woman's friend brought in to testify by defense that woman is sexually promiscuous. Can this be objected to?

The evidence will be excluded, because it violates the rape shield law. Discussion of correct answer:In a criminal case, barring certain limited exceptions, evidence of a rape victim's prior sexual conduct or sexual predisposition is inadmissible. Pursuant to these exceptions, such evidence is admissible to prove that a person other than the accused is the source of semen, the victim's injury, or other physical evidence, or if exclusion of the evidence would violate the constitutional rights of the defendant. Specific instances of sexual behavior between the accused and the victim may also be offered to prove consent. Because none of these exceptions apply to the proposed testimony, it is inadmissible. Incorrect. The evidence will be excluded, because the evidence is unfairly prejudicial. The Federal Rules of Evidence specifically address the issue of when evidence of the prior sexual conduct of a victim of a sex crime is admissible. Pursuant to the "rape shield law," barring certain limited exceptions, evidence of a rape victim's prior sexual conduct is generally inadmissible. Thus, while it may be true that the friend's testimony might be unfairly prejudicial, the better analysis of the admissibility of this evidence would take into account the particular provisions of the rape shield law. Therefore, this is not the best answer.

Which of the following statements describes the effects of taking judicial notice in a civil suit?

The fact judicially noticed is conclusively established. Discussion of correct answer:According to Rule 201(g) of the Federal Rules of Evidence, the effect of taking judicial notice in a civil suit is that the judicially noticed fact is conclusively proven. Choices (B) and (C) are both incorrect as neither the burden of persuasion nor the burden of producing evidence is shifted when a court takes judicial notice in either a civil or a criminal case. As noted in choice (A), the fact is conclusively established in civil cases. In a criminal case, the prosecution always has the burden of proving each element of the crime beyond a reasonable doubt. Although the taking of judicial notice will satisfy the prosecution's burden, there is no shifting of the burden to the criminal defendant. Choice (D) is incorrect. According to Rule 201(b), a judicially noticed fact must be one which is not subject to reasonable dispute because it: (1) is generally known within the trial court's territorial jurisdiction; or (2) can be accurately and readily determined from sources whose accuracy cannot reasonably be questioned. If the other party is able to offer evidence contrary to the fact that the court is considering taking judicial notice of, then the fact would be subject to reasonable dispute and would not fall within the requirements of Rule 201(b).

Who decides if it was a valid "Dying Declaration?"

The judge, without assistance from the jury. Discussion of correct answer:This is the best choice because Federal Rule of Evidence 804(b)(2) provides for the admissibility of a dying declaration where: (1) the declarant is unavailable; (2) the declaration was made while the declarant believed his death was imminent; (3) the declaration concerned the cause or circumstances of what the declarant believed to be his imminent death; and (4) the declaration is offered in either a homicide prosecution or a civil proceeding. The FRE broadened the traditional use of dying declarations in that the declarant need not die (only be unavailable) and the declaration can be used in civil actions. The above facts would allow the judge to admit the statement as a dying declaration. It is solely the judge's role to determine whether a proffered out-of-court statement is offered for a non-hearsay purpose, and, if not, whether any hearsay exceptions apply (such as whether the mechanic believed his death was imminent). The jury then determines the weight and credibility of the evidence.

Mandatory Presumption's in statute's are problematic

The jury is permitted, but not required, to infer that the guns were in the dancer's possession, and the burdens of production and persuasion are not affected. Discussion of correct answer:A presumed fact is a fact whose existence can be inferred from the existence of a basic fact. In criminal trials, the application of evidentiary presumptions raises constitutional issues. As the Federal Rules of Evidence do not address the use of presumptions in criminal trials, the rules covering the use of presumptions in criminal trials come from federal case law. In a criminal context, a presumption is referred to as "permissive" if the jury is instructed that it may infer a presumed fact from the existence of a basic fact. There are also mandatory presumptions requiring the presumed facts be inferred. Generally speaking, the use of a mandatory presumption in a criminal trial will be found unconstitutional because every element of a crime charged must be proved beyond a reasonable doubt to convict a criminal defendant. Therefore, presumptions used in a criminal trial should be interpreted as having the effect of a permissive presumption. Thus, here, as a result of the permissive presumption, the jury is permitted, but not required, to infer that the guns were in the dancer's possession. The burdens of production and persuasion are not affected. Incorrect to say that the jury must infer that the guns were in the dancer's possession, unless the dancer produces evidence showing that the guns were not in his possession. This answer describes a type of mandatory presumption, which affects the burden of producing evidence. Case law is unclear as to whether the use of this type of mandatory presumption is constitutional in a criminal trial but generally, the Sixth Amendment guarantees an accused the right to a jury trial and all elements of a crime must be proved beyond a reasonable doubt. However, the use of permissive presumptions is constitutional, as long as the jury could reasonably infer the presumed fact from the basic fact that gives rise to the presumption. A permissive presumption allows, but does not require, the jury to infer the presumption once the basic facts giving rise to the presumption are established, but it does not affect the burden of producing evidence nor does it affect the burden of proof. Presumptions in criminal cases, such as the presumption at issue in this question, should generally be interpreted as permissive presumptions that affect neither the burden of producing evidence nor the burden of proof.

Recording of salesman making pitch. Which is the least sufficient basis for admitting the recording?

The lay person's testimony that he had heard the same voice on a similar tape recording identified to him by the salesman's brother as being the voice of the salesman. Discussion of correct answer:The problem of authenticating oral communications that are heard (as opposed to writings, which are seen) is analogous to the authentication of writings. As a general rule, a document is not admissible unless it has been authenticated (i.e., sufficient proof offered that the document is what it purports to be). Federal Rule of Evidence 901 provides that a party may authenticate a document by admission or stipulation of the opponent, by use of a witness, by jury comparison, or by circumstantial evidence. Authentication that a recorded voice is that of a particular speaker may come from testimony that the witness is familiar with the voice of the person alleged to be the speaker. Thus, the testimony of a lay person who was not familiar with the salesman's voice, had never heard it in person and believed that the recording was of the salesman's voice only because another person identified it as such, does not demonstrate sufficient personal knowledge of the fact to which he would testify. As such, this would not be an acceptable basis upon which to authenticate a second recording as being of the salesman's voice. Incorrect. The lay person's testimony that he had talked with the salesman over the phone at a time after the recording was made to prepare to testify at trial. A witness may testify to the identity of a recorded speaker if the witness met and talked to the speaker for the first time after the recording was made. With regard to authentication of a voice recording, the issue is whether the witness is familiar enough with the voice on the recording to be able to testify that the recording is of the person that the proponent of the evidence purports asserts is the case. That the witness developed that familiarity after the recording was made does not affect his ability to authenticate a recording as being the voice of a particular person.

An officer should not identify evidence based purely on a brand

The officer testifies that he recognizes the bottle because it is the same brand as the one the arresting officer took from the farmer. Discussion of correct answer:There are only three manners in which a physical object can be authenticated--by testimony of personal knowledge which shows familiarity with the object to be authenticated, by distinctive markings or characteristics, or by chain of custody. This choice does not present a scenario fitting into any of those categories. Based on the witness's testimony, there is no way for the witness to distinguish this liquor bottle from any other of the same brand.

The Privilege for Marital Communications

The privilege for marital communications extends to any confidential statement made between spouses during the existence of a legal marriage. When the privilege applies, either spouse can prevent the other from disclosing such confidential marital communications. In contrast to the spousal privilege, the marital communication privilege protects only communications made during, not before, a valid marriage. In this case, the statements at issue were made before the real estate agent was married. Therefore, the statements will not be protected by the marital communication privilege.

cameras did not show story of def pan out. What additional character evidence can be brought in?

The prosecution calls the arresting from a previous and unrelated case nine years ago where the defendant was convicted of armed robbery. Discussion of correct answer:Evidence of a prior conviction may be introduced for impeachment purposes if it is a misdemeanor involving dishonesty or if it is a felony. This general rule is subject to a few exceptions. One of those exceptions is that for any felony that does not involve dishonesty, the probative value of the conviction must outweigh its prejudicial effect. When the conviction is for a serious felony identical to the one with which the defendant is being charged, there is little question that there is a very strong prejudicial effect that outweighs the limited probative value that the conviction offers with respect to the defendant's truthfulness. Incorrect. The prosecution calls the defendant's mother to testify. The mother testifies that the defendant is well known in the family for being a liar. A witness's credibility can be impeached the appropriate use of opinion and reputation evidence or by evidence of specific instances which resulted in a criminal conviction or which are inquired about on cross-examination. Here the mother is being asked on direct examination about the defendant's reputation for honesty. Because the defendant put his veracity at issue by taking the stand, this testimony is admissible.

Trick Question: Is it a sentencing hearing?

The rules of evidence do not apply. Discussion of correct answer:The Federal Rules of Evidence do not apply to sentencing hearings. For purposes of sentencing the defendant, the judge may consider hearsay and other evidence that would not be admissible at trial. Although the wife's statements are hearsay, the Federal Rules of Evidence do not apply, and the husband may tell the court what the wife said. A hearsay exception exists for the declarant's then-existing mental, emotional, or physical condition. This exception does not include statements of memory or belief to prove the fact remembered or believed. The facts do not state enough about the contents of the wife's statements to determine whether this hearsay exception would apply to her statements. However, it is irrelevant whether the hearsay exception applies because the Federal Rules of Evidence do not apply to sentencing proceedings.

Wheeled into operating word after being told that he was likely to die, so he said that he had stopped the engine to his wife before going in. Admissible?

The statement is admissible as a dying declaration. Discussion of correct answer:Choice (C) is incorrect because according to F.R.E. 804(b)(2), dying declarations are admissible in homicide prosecutions or in civil actions. Thus, dying declarations are admissible in all cases except non-homicide criminal cases. A second limitation to the dying declaration rule is the statement must refer to the cause or circumstances attending the supposed impending death. Thus, a "deathbed" statement by one indicted for a bank robbery that his co-indictee was innocent of the crime charged fell outside the present exception because it did not describe the cause or circumstances of the declarant's impending death. In contrast, the worker's statement did concern the circumstances of what he believed to be his impending death. Therefore, it would qualify as a dying declaration, and choice (A) is correct.

Daughter is bittern by goat at zoo, guy at zoo offers $100 and a sorry. Admissible?

The statement is admissible, as an admission. Discussion of correct answer:Of the MBE subject areas, the Evidence questions are the trickiest. Here is a very subtle distinction that is often tested. Many students will mistakenly choose choice (D) thinking that this is a settlement or compromise offer. In order to exclude statements made in compromise negotiations, there must be an actual dispute or, at least, an apparent difference of view between the parties as to the validity or amount of a claim. In this particular question, the testmaker is concluding that no dispute existed at the moment the zoo owner offered to pay the daughter the $100. On the contrary, the zoo owner is making a spontaneous statement immediately following the incident at the petting zoo.

Another journal article, this time contradicting prosecution's expert doctor. Is the statement making the contradiction admissible and as what?

The statements may be read to the jury by the witness but not received as exhibits. Discussion of correct answer:While hearsay is generally inadmissible, there are numerous exceptions available which allow it to be admitted in court and offered to prove the truth of the matter asserted. One such exception is the learned treatises exception to the hearsay rule. This exception states that to the extent called to the attention of an expert witness upon cross-examination, or relied upon by the expert witness in direct examination, statements contained in published treatises, periodicals, or pamphlets relating to history, medicine, science, or art, may be read into evidence but not received as exhibits if they have been determined to be reliable authority by testimony, admission of the witness, other expert testimony, or judicial notice. Here, the witness was qualified as an expert, the expert established the authority of Medicine Today, the article was admitted, and then presented to the expert on cross-examination.

Biker's witness offered testimony to discredit plaintiff witness, that he wasn't at the scene. Which of the following forms of evidence would allow the pedestrian to introduce evidence of the store owner's reputation for truthfulness?

The store owner was out of the state at the time of the incident. Discussion of correct answer:Evidence intended to support the credibility of a witness may generally not be admitted unless the credibility of that witness has previously been attacked (in such a situation, the evidence is said to "rehabilitate" the impeached witness). Moreover, the mere fact that the opposing party has presented other witnesses or evidence that contradicts the testimony of a witness is insufficient to justify an attempt to rehabilitate or bolster the credibility of a witness. When a witness has been impeached, the party originally calling the witness may then seek to rehabilitate the credibility of the attacked witness. Evidence that the store owner was not present at the accident scene would be an attack on his character for truthfulness, because it directly contradicts the store owner's testimony that he observed the biker's failure to yield. Therefore, to counter such evidence, it would be proper for the pedestrian to offer evidence of the store owner's good character for truthfulness, to rebut the implication that the store owner lied about having observed the collision. Incorrect. The store owner used to be in love with the pedestrian. Evidence of a witness's character for veracity is admissible for purposes of rehabilitating a witness whose veracity has been impeached. While evidence of the store owner's past relationship with the pedestrian might tend to show that he is biased and might testify falsely on her behalf, this evidence might also be taken to show just the opposite, that the store owner would be more likely to falsely testify against the pedestrian out of hurt or revenge, thereby serving to strengthen the credibility of the store owner's testimony. Given that evidence that the store owner had been in love with the pedestrian does not directly bear on whether the store owner's testimony is true, evidence of the store owner's character for veracity would be more appropriate to counter direct evidence that the store owner's testimony could not possibly have been true. As such, this is not the best answer.

Asking the Def. to roll up your sleeves so we can see those needle marks. Allowed?

The student must comply, because the evidence is relevant to her credibility. Discussion of correct answer:This evidence, which could indicate recent drug use, is relevant to attack the student's assertion she has not used drugs in more than two years. Because she was charged with possession of drug paraphernalia and marijuana, her drug use is an important issue in the case.

Officer made video of confession. Officer proposes to testify about confession. Is this allowed?

The testimony is admissible, because the officer had firsthand knowledge of the confession. Discussion of correct answer:The best evidence rule applies only when the proponent is attempting to prove the contents or terms of a writing. Note that sometimes a writing recites or records a perceivable event or condition such as a marriage (marriage certificate), payment of money (receipt), or the utterance of certain words (transcript). Here, the proponent wishing to prove the underlying event may offer testimony as an observer. This does not involve the best evidence rule because the proponent is not attempting to prove the terms of a writing, but merely is presenting evidence of an event perceived by a witness with firsthand knowledge. In this hypo, the officer's testimony is not reliant on the writing or the videotape. He had firsthand knowledge of the suspect's confession because he was present and overheard it. Incorrect. The testimony is inadmissible, because the videotape is the best evidence of the suspect's confession.

Which person would best lay foundations for the photo of the defendant's fencing?

The woman's neighbor, who has lived next door to the woman for 20 years and was sufficiently familiar with the woman's property. Discussion of correct answer:Pursuant to Federal Rule of Evidence 901, tangible evidence may be admitted when it has been authenticated. Authentication of a photograph entails showing that the photograph represents what it purports to represent. Pursuant to Federal Rule of Evidence 901(b), when a photograph is used, some person must testify that he or she is familiar with the scene depicted in the photo and that the photo is an accurate depiction of the scene. In most cases, it is not required that the person testifying be the person who actually took the photo. Here, in order to authenticate that the photograph represents what the woman's property looks like, it must be established that the scene has not been substantially altered since the time the photograph was taken. As long as the neighbor is sufficiently familiar with the woman's property in its present condition, he can authenticate the photo by testifying that the photo accurately depicts the woman's property. Incorrect. The person who took the photo. Pursuant to Federal Rule of Evidence 901, tangible evidence may be admitted when it has been authenticated. Authentication of a photograph entails showing that the photograph represents what it purports to represent. Pursuant to Federal Rule of Evidence 901(b), when a photograph is used, some person must testify that he or she is familiar with the scene depicted in the photo and that the photo is an accurate depiction of the scene. In most cases, it is not required that the person testifying be the person who actually took the photo.

Manufacturer offers settlement before trial. What is a proper purpose for use of such by plaintiffs?

To establish an effort by the manufacturer to obstruct a potential criminal investigation. Discussion of correct answer:While evidence of an offer to settle a claim is generally inadmissible in court, admission of such evidence may be permitted for the limited purposes of proving a witness's bias or prejudice, for negating a contention of an undue delay, and for proving that an effort exists to obstruct a criminal investigation or prosecution. Incorrect. To impeach a representative of the manufacturer through a prior inconsistent statement. Although settlement offers and offers to compromise may be admissible for some specific purposes, they may not be admitted in an effort to impeach a witness through prior inconsistent statements. This would not be a proper basis to present settlement information.

Witness saw the trucker lumber drunkenly, looking scared, with a limp. What is not an appropriate consideration in determining whether the witness can testify to these opinions?

Whether the witness's testimony is the product of reliable principles and methods. Discussion of correct answer:This choice is improper because it sets forth a standard which is only to be applied when an expert witness is testifying. When testimony is being given by an expert, the court must determine whether the testimony is based on reliable methods or principles. However, the same standard does not apply to a lay witness, making this an improper consideration. Here, there is nothing in the facts that would indicate that the witness was tendered as an expert by either side at the trial. Therefore, we can assume that she is testifying as a lay witness. Lay witnesses are able to provide their opinions in certain circumstances. The proper scope of opinion testimony given by a lay witness includes but is not limited to perceptions like speed, measurements, physical states like intoxication or injury, personal emotions, and sensory descriptions like smell. Incorrect. Whether or not the testimony is based on scientific, technical, or specialized knowledge. A lay witness is able to provide testimony where she gives an opinion or draws an inference if that opinion is rationally based on the perception of the witness, if the testimony is helpful in gaining a clear understanding of her testimony or in determining a fact in issue, and if that opinion testimony is not based on scientific, technical, or other specialized knowledge which would fall outside of the scope of the lay witness's knowledge. No specialized knowledge is required in determining whether a person looked drunk, afraid and hurt. Here, the neighbor limited her testimony only to that which was within the permissible scope of lay witness testimony and thus, this would be a proper consideration.

Hospital admin said plaintiff was not conscious when she got to hospital. is this admissible?

Yes because a lay opinion as to a physical state is admissible. Discussion of correct answer:The Federal Rules of Evidence permit a lay witness to testify regarding an opinion if the opinion is based on the witness's rational perception of events and helps give a clearer understanding of the testimony. The proper scope of non-expert opinions includes perceptions of physical states, including intoxication or injury. Here, the hospital administrator is testifying as to her opinion that the flight attendant was unconscious at the time she arrived at the hospital. Because consciousness is a physical state, the hospital administrator's opinion testimony regarding the flight attendant's physical state at the time of her admission to the hospital should be admitted. Therefore this answer is correct. Incorrect. Yes, because it was a present sense impression. Hearsay evidence is an out-of-court statement offered to prove the truth of the matter asserted and is generally inadmissible unless an exception applies. The Federal Rules of Evidence establish an exception to the hearsay rule for a witness's statement explaining or describing an event and made at or immediately after the time of the event. This provision is known as the "present sense impression" exception to the hearsay rule. In this case, the hospital administrator is testifying to her opinion of the flight attendant's state of consciousness at the time of her arrival at the hospital; there is no statement being offered into evidence that was made at the time of or immediately after the event. Therefore, the present sense impression exception does not apply and this answer is incorrect.

Witness layman said he saw car going 60 mph. Admissible?

Yes, as a proper lay opinion, which is relevant to help the trier of fact prove a fact in issue. Discussion of correct answer:In accord with Federal Rule of Evidence 701, if the witness is not testifying as an expert, the witness' testimony in the form of opinions or references is limited to those opinions or inferences that are: (1) rationally based on the perception of the witness; and (2) helpful to a clear understanding of the witness' testimony or the determination of a fact in issue.

Testimony of a bystander who heard the daughter say that the boat owner would be mad because she took it without his permission. Boat owner seeks to introduce. Admissible?

Yes, as a statement against interest. Discussion of correct answer:This statement could only come in as a statement against interest. Under Federal Rule of Evidence 804(b)(3), a statement against interest requires that: (1) the declarant be unavailable at trial; (2) the statement was contrary to the declarant's pecuniary, proprietary, or penal interest, or tended to subject the declarant to liability when it was made; and (3) a reasonable person in the declarant's position would not have made the statement if it was not true. This statement would have tended to subject the friend's daughter to criminal or civil liability for theft or conversion, as well as damages for the accident. The statement of the friend's daughter will fall under the statement against interest exception to the hearsay rule. The statement would tend to subject the friend's daughter's to criminal or civil liability (for theft or conversion) as well as damages for the accident, and she is now unavailable to testify at trial, as required by Rule 804.

Government asks bookkeeper of defendant for ledger showing cash admissions. Admissible?

Yes, as a statement by an opposing party. Discussion of correct answer:In the physician's criminal prosecution for income tax evasion, the document offered by the government will be admissible as a statement by an opposing party. Choice (B) is correct because the document has been properly authenticated by other evidence, establishing that it was written in the physician's handwriting, and the ledger of cash receipts is highly relevant to the cause of action of income tax evasion. Choice (C) is incorrect because statements are defined as nonhearsay under the Federal Rules of Evidence. Choice (D) is incorrect because the Fifth Amendment privilege against self-incrimination applies only to evidence that is testimonial in nature (i.e., not a ledger). Choice (A) is very close, but not the best answer. It has not been established in the facts that the document was prepared in the ordinary course of business or that it was the regular practice of the physician to keep such a separate ledger for unreported cash receipts. The government tried to lay the necessary foundation for the business record by asking the physician to describe his system of bookkeeping, but he refused to testify. Nevertheless, the ledger will still be admissible as a statement by an opposing party.

An insurance company seeks to deny plaintiff because he may have killed victim. Is victim's mother's testimony during trial admissible?

Yes, as former testimony. Discussion of correct answer:Under Federal Rule of Evidence 804(b)(1), prior testimony given in any proceeding, such as a deposition, trial, etc., can be admitted as an exception to the hearsay rule. Such testimony can be admitted as long as the witness is unavailable to testify, and death would certainly exclude her from doing so. The other condition is that the party against whom the testimony is offered must have had a chance to develop the witness' testimony by cross-examination, direct, or redirect. The party must also have had a similar motive in developing the testimony. The mother is deceased, but the friend would attack her testimony in the second trial for a similar motive as in the first; that is, he is trying to prevent a jury from concluding that he had murdered his friend. He had the chance to examine the mother in the first trial, so all of the conditions are met. Her testimony is admissible.

Patient told doctor that she had been hit in the back by defendant and had not recovered from earlier back problem. Can Def bring testimony of doctor in?

Yes, as nonhearsay. Discussion of correct answer:This is a tricky Evidence question. Students must be aware that the bar examiners often test the rules about opposing party's statements--statements of a party offered by the opponent party. Here, the question attempts to lead students toward the hearsay exception for statements made for medical diagnosis or treatment. But, it is unnecessary to consider a hearsay exception, because opposing party's statements are nonhearsay and freely admissible. Students would do well to approach hearsay questions by first asking if the declarant's statement is one of a party that is being offered by the opponent. If so it is admissible without reliance on a hearsay exception.

Employment firm offered woman and woman stole. Plaintiff seeks to offer evidence the woman had stolen from an earlier position. Admissible?

Yes, as proof of the woman's character trait for dishonesty. Discussion of correct answer:This is a civil action in which character is in issue. The agent represented that the woman, his employee, was "honest and trustworthy." After the woman was fired for stealing, the real estate company sued the staffing agency for negligent hiring. Remember that in negligent hiring cases, it is the character of the employee, not the employer that is in issue. In this situation, the woman's character for honesty is in issue, not the staffing agent's and therefore (C) is incorrect. Under F.R.E. 405, "in cases in which character or a trait of character of a person is an essential element of a charge, claim, or defense, proof may be made by specific instances of that person's conduct." As a general rule, character is usually "in issue" in negligent hiring actions. Choice (A) is incorrect because this question does not involve impeachment but rather deals with proof of character where a person's character trait is in issue. Choice (D) is incorrect because the Federal Rules of Evidence allow specific instances of misconduct as proof of character in cases in which character is in issue.

So... a crack addict is an expert to testify on crack. The more you learn.

Yes, as proper expert opinion. Discussion of correct answer:Because of the addict's longtime use of cocaine, he can qualify as an expert witness. Formal education is unnecessary to attain status to render an expert opinion under the Federal Rules of Evidence. Rule 702 requires that: (1) the witness be qualified as an expert by knowledge, skill, experience, training or education; (2) the scientific, technical, or other specialized knowledge possessed by the witness will assist the jury; and (3) the witness is able to testify with a reasonable degree of probability concerning the opinion. The addict meets those tests.

Crooked Politician claims he's an honest politician, can witness testify he took a bribe?

Yes, as relevant evidence of the mayor's character as a corrupt politician. Discussion of correct answer:When a person's character is in issue, character evidence is admissible in all forms (reputation, opinion, and specific instances of conduct). Here, the mayor's character as a corrupt (or honest) politician is in issue, because it was the subject of the defamatory statement and the defense of truth is asserted. It is not impeachment that is at issue here, but rather, the mayor's character. Therefore, this evidence is admissible because it is being used for substantive proof of the truth or falsity of the defamatory statement.

Plaintiff sued journal for defamation (cheating IRS). defendant seeks to bring in evidence the plaintiff lied on tax return. Admissible?

Yes, as substantive evidence of the plaintiff's character. Discussion of correct answer:The character evidence about the plaintiff distorting the value of certain investments is admissible because his character as a "tax chiseler" is at issue under the truth defense in this defamation case. This is one of the instances where, in a civil case, evidence of specific bad acts may be used to prove a character trait. Generally, character evidence is not admissible in civil trials unless it is impeachment evidence in the form of reputation or opinion evidence about a witness's truthfulness, or a question asked of the witness herself about prior bad acts relating to truthfulness. In all the examples just listed, the evidence is used to attack the credibility of the witness only; and does not come in substantively. However, in a defamation action such as this one the issue of a person's character is an essential element of a charge, claim, or defense. Therefore, under Rule 405(b) of the Federal Rules of Evidence, proof of character by using specific instances of conduct is permitted. Reputation or opinion evidence, though not offered here, would also be allowed to show that the plaintiff cheated on his taxes, because all three types of character evidence may be used to prove the essential character issue in this defamation case. Choice (D) is incorrect because corroboration is not required before admitting proper character evidence.

So she said she heard someone say that her friend told her that, like, she threw a box out the women onto the plaintiff (that bitch!)... Can this be admitted if only the first one is available?

Yes, because Susan's statement, although double hearsay, was admissible under exceptions to the hearsay rule. Discussion of correct answer:Federal Rule of Evidence 805 provides that hearsay included within hearsay is not excluded under the hearsay rule if each part of the combined statement conforms with an exception to the hearsay rule. Here, Julie's statement to Susan would have been admissible as a statement against interest pursuant to Federal Rule of Evidence 804(b)(3), as Julie is unavailable to testify due to her suicide, and her actions would have subjected her to civil and criminal liability. Susan's statement would likewise be admissible, as an excited utterance, defined pursuant to Federal Rule of Evidence 803(2) as a "statement relating to a startling event or condition made while the declarant was under the stress of excitement caused by the event or condition." Finally, the evidence that Julie dropped the box on Donna is relevant to show that Streamline is not liable for Donna's injuries. Therefore, Catherine's testimony is admissible. Incorrect. Yes, because Julie's statement is a statement of an opposing party's employee. Federal Rule of Evidence 801(d)(2) provides that an statement by an employee of a party is not hearsay and is admissible. However, for this rule to apply, the statement must be made while the person is employed with the company, and it must be made on a matter within the scope of that relationship and while it existed. Here, Julie was an employee of Maggie's. However, Federal Rule of Evidence 801(d)(2)(C) provides that the statement by the agent or servant must concern a matter within the scope of his agency or employment, made during the existence of the relationship. Julie's tortious and criminal conduct is obviously not within the scope of her employment. Therefore, Julie's statement, that she dropped the box on Donna while employed at Maggie's, is not a statement of an opposing party.

The specialist proposes to testify that the plaintiff's injuries were caused by the defective design of the lighter, and not the gas leak based on reports on the case. May he?

Yes, because experts in this field may rely upon the same type of reports and materials in reaching this type of conclusion. Discussion of correct answer:This is the correct answer because Federal Rule of Evidence 703 permits an expert to base her opinion testimony on facts perceived by her or made known to her at or before the hearing, which, if the facts are of a type reasonably relied upon by experts in the particular field, need not be admissible in evidence at the hearing. This choice accurately states this standard for the basis of the expert opinion. Under Federal Rule of Evidence 704 and the modern rule, an expert's opinion testimony is not inadmissible simply because it relates to an ultimate issue. This states a rule followed only in some traditional jurisdictions.

Witness is 4, and now 6. May Witness child testify?

Yes, because he had personal knowledge of the accident. Discussion of correct answer:Under FRE 601, every person is competent to be a witness. FRE 602 limits witnesses to testifying as to matters within their personal knowledge. A witness with personal knowledge of the accident will be competent to testify, even if that witness is a child. At the common law, witnesses would be competent only if they both possessed personal knowledge of the subject matter of their testimony and possessed the capacity to perceive and tell the truth. However, these latter requirements have been abandoned by the FRE. Under the FRE, questions of mental competence (based on age, illness, drug use, etc.) go to the weight of the evidence, and not admissibility. Thus, the child is a competent witness in this action, and this answer choice is correct. Incorrect. No, if he lacked the capacity to perceive and relate the accident at the time of its occurrence. This answer choice states the applicable rule under the common law, not under the FRE. The FRE provides that any witness with personal knowledge is competent to testify. Questions as to the witness's mental competence (such as those based on age, illness, drug use, etc.) go to the weight of the testimony, not its admissibility.

Naive store owner want to bring in purchase sheet in an attempt to show defendant had taken more than she paid for. Is this relevant?

Yes, because his testimony relates to relevant material that bears on a material aspect of the suit. Discussion of correct answer:Federal Rule of Evidence (FRE) 401 provides that "relevant" evidence means evidence having any tendency to make the existence of any fact that is of consequence to the determination of the action more probable or less probable. FRE 602 provides that a witness may not testify to a matter unless evidence is introduced sufficient to support a finding that he has personal knowledge of it. Here, the store owner has personal knowledge of the items that were not listed on the customer's sales sheets but that were missing from the store following her visits there. Therefore, the store owner's testimony is relevant and admissible. Incorrect. Yes, because the customer "opened the door" when she introduced into evidence the sales sheets. A party is considered to have "opened the door" when he offers testimony on a certain subject and cannot prevent another party from offering rebuttal testimony on the same subject. However, it is not necessary to rely on this theory. In this case, the sales sheets and the store owner's shortages are relevant to the ultimate issue of whether the customer owed the store owner $300 or $700. The testimony would be relevant even if the store owner, rather than the customer, had raised the issue of the sales sheets' accuracy. Therefore, there is no need for the customer to "open the door" to this issue and this answer is not the strongest ground for admitting the store owner's testimony.

Can a nolo contendere plea be used to prove part of a later prosecution case on an unrelated subject matter?

Yes, because it is an essential element of the proof of the crime that must be established. Discussion of correct answer:Under Federal Rule of Evidence 404(b), a defendant's past crimes can be used to prove the circumstantial element of the charged crime, such as intent. (This is one of the MIMIC issues.) To show that the bar owner has made a false statement to the liquor board, the board must show that he had a previous felony conviction. Federal Rule of Evidence 410(a)(2) states that evidence of a nolo contendere plea is not admissible against the defendant who made the plea or participated in the plea discussion. However, courts have interpreted this to mean that such a plea is inadmissible to prove that the defendant is guilty of the crime charged (because a nolo contendere plea is not an admission of guilt), but would still be admissible to prove the fact of the previous conviction [United States v. Adedoyin, 369 F.3d 337 (3d Cir. 2004)]. The bar owner is charged with making a false statement about his prior conviction. Thus, evidence of the fact of his prior conviction can be admitted--his plea cannot be used to show that he committed the assault of a patron, but it can be used to show he made a false statement about having a previous felony conviction. It is true that Federal Rule of Evidence 404(a)(2) prohibits the admission of evidence of a nolo contendere plea against a defendant. However, courts have interpreted this to mean that such a plea is inadmissible to prove that the defendant is guilty of the crime charged (because a nolo contendere plea is not an admission of guilt), but would still be admissible to prove the fact of the previous conviction [United States v. Adedoyin, 369 F.3d 337 (3d Cir. 2004)]. Thus, evidence of the nolo contendere plea can be admitted to prove the fact of the previous conviction, though not the bar owner's guilt of the assault charge.

Tenant seeks to introduce testimony of what he told the landlord about the trees before the incident. Admissible?

Yes, because it is not hearsay being offered against the landlord. Discussion of correct answer:The tenant is using the statement as a statement of warning, rather than substantively for its truth. Therefore, it is not hearsay evidence, and will instead be is admissible to show that the landlord was warned of the dangers of the trees. Incorrect. No, because the tenant's testimony is hearsay. Hearsay is inadmissible unless there is an exception allowing for its admissibility. However, this evidence is not being offered for its truth. Rather, the tenant is using the statement as one of warning, to let the landlord know that the trees looked weak, or were succeptible to falling over.

An offer after trial starts by defendant for witness to have a armory lapse. Admissible?

Yes, because it is probative of the investor's consciousness of guilt. Discussion of correct answer:This is admissible to show consciousness of guilt, because the jury may infer the investor would not try to silence a witness if he were not guilty and fearful of that witness's testimony. The rule barring an offer of settlement as an admission of guilt does not apply the same way here as it normally would because this type of "settlement" is not encouraged by social policy--in fact, it is witness tampering.

Store owner says he say plaintiff cross all the time carefully. Admissible?

Yes, because it is relevant as to whether the pedestrian crossed against the red light on this occasion. Discussion of correct answer:According to Federal Rule of Evidence 406, evidence of habit is relevant to prove the conduct of a person on a particular occasion. This testimony of the newsstand owner is sufficient because he observed him for three years, and the pedestrian never went across the street when the light was red. "Never" or "always" are key words to establish habit. Incorrect. Yes, to show that the pedestrian had a reputation for acting safely. This choice is incorrect because the evidence is offered to prove the pedestrian's conduct conformed to his habit--not reputation. The information given does not show anything about the pedestrian's behavior elsewhere or the community opinion of him.

Court ruled husband could testify as to what he saw in presence of third person. Affirm this setup?

Yes, because only the husband as the witness spouse can assert the spousal privilege. Discussion of correct answer:The spousal privilege can be asserted to protect a spouse from having to testify against the other spouse. Federal courts as well as most state courts now follow the U.S. Supreme Court's ruling in Trammel v. California [445 U.S. 40 (1980)] that the witness spouse, rather than the party spouse, holds the privilege and can choose whether to assert the privilege to avoid testifying. In this case, the husband may testify against the wife if he chooses to do so, and the wife cannot assert the spousal privilege. Therefore this answer is correct. Incorrect. No, because the wife can assert the spousal privilege to block the husband's testimony. This answer is incorrect because the U.S. Supreme Court has ruled that the spousal privilege can be asserted by the witness spouse, not the party spouse. Federal courts and most state courts now follow that U.S. Supreme Court ruling. Therefore, it is unlikely the wife can assert the spousal privilege to prevent the husband from testifying.

Nurse refreshed herself with records kept by the secretary. Did not have records present while testifying, did not seek to have them admitted. Is this allowed?

Yes, because she merely used the records before testifying to refresh her recollection regarding the nature and extent of the secretary's injuries. Discussion of correct answer:The Federal Rules of Evidence permit a witness who is unable to recall an event to use a writing, photograph, or other evidence to help refresh her memory to recall the event, so long as a proper foundation for that evidence is provided. The item being used to refresh the witness's memory may not necessarily be entered into evidence, but it must be made available to the adverse party for inspection. The witness cannot keep the evidence present while testifying, but may look to the evidence before testifying to refresh her recollection. In this case, the nurse's testimony can be admitted because she is merely using the medical records of the secretary's injuries to refresh her memory. Therefore, this answer is correct. Incorrect. Yes, because it is a past recollection recorded. The Federal Rules of Evidence permit admission of evidence in the form of writing of a past recollection recorded if the witness has a personal knowledge of the matter but cannot now recall it. To be considered a past recollection recorded, the writing must have been made at or near the time of the event. Here, the recorded recollection is not what is sought to be introduced but rather the nurse's testimony. She is merely using the document to refresh her recollection of the events. Additionally, while it is possible that the medical records were made near the time of the secretary's injuries, it is not clear whether the nurse personally made them. Therefore, because the nurse is not testifying about her recorded past recollection, but about what she recalls about the secretary's injuries, the rules of evidence regarding past recollections recorded are not applicable, and this answer is incorrect.

Witness will testify she (as roommate) saw tax driver from crash drunk a car many times. Will cab company fail objection?

Yes, because the cab driver's character is a material issue in the litigation. Discussion of correct answer:Federal Rule of Evidence 405 holds that when the character of a person is a substantive issue in the litigation, evidence of his character is directly relevant and is admissible in any form: reputation, opinion, or specific instances of conduct. Here, the basis of the mother's complaint is that the cab company was negligent in hiring the driver and entrusting a cab to him while failing to discover that the driver drove motor vehicles while intoxicated. Thus, the matter of whether the driver in fact drove motor vehicles while intoxicated is a material issue of the case, and evidence that he had done so on previous occasions is admissible as directly relevant to that issue. Incorrect. No, because the cab driver has not testified that he has never operated a motor vehicle while intoxicated. This answer attempts to misdirect students by suggesting that impeachment of the driver is the only permissible use for the proffered character evidence. The roommate's character evidence is not offered to impeach the driver or any defense witness; rather, this evidence is relevant to a determinative issue in the case (whether the driver was an alcoholic and tended to operate motor vehicles while intoxicated). Thus, the evidence is relevant and admissible, regardless of whether the driver himself has testified that he has never operated a motor vehicle while intoxicated.

Witness issued statements outside scene identifying culprit to friends. Witness is at the courthouse now. Are the statements admissible?

Yes, because the college student's statement identifying the man is not hearsay. Discussion of correct answer:Hearsay is an out-of-court statement offered to prove the truth of the matter asserted. There are some exemptions, however, to the hearsay rule, such that certain out-of-court statements are not hearsay, even if offered for the truth of the matter asserted. One exemption is where the declarant is a witness, and the statement is a prior identification. The college student, the declarant, is a witness, and the statement was a prior identification of the man. Therefore, the statement is not hearsay, and it is admissible for that reason. There are certain hearsay exceptions that apply only when the declarant is unavailable, including former testimony, a statement under the belief of imminent death, a statement against interest, a statement of personal or family history, and a statement offered against a party that wrongfully caused the declarant's unavailability. However, none of these exceptions is relevant. The availability of the college student (the declarant) is relevant because a statement is not hearsay when the declarant is a witness, and the statement was a prior identification. Because the college student is available and is a witness, the prior identification is not hearsay and is admissible at trial.

Father arrested for battery, seeks to testify he is honest. Sustain objection against by Prosecution?

Yes, because the defendant's veracity is not pertinent to the crime of battery. Discussion of correct answer:The general rule is that evidence regarding a person's character is inadmissible to prove he has acted in conformity with that character. Furthermore, the prosecution may not introduce evidence of a defendant's bad character to show he is the type of person to commit the crime charged. But one exception to the general rule regarding character evidence is that the defendant may introduce evidence of his own good character traits if those good character traits are inconsistent with the crime charged. Here, the defendant seeks to testify that he is honest to support his testimony regarding his whereabouts during the fight. However, the defendant is charged with battery, and the elements of that crime have no bearing on the defendant's truthfulness. Thus, the defendant's testimony regarding his own veracity is inadmissible. Incorrect. No, because a defendant may always offer evidence of his good character in a criminal case. This answer incorrectly states the law. A defendant may offer evidence of his good character traits only where those traits are inconsistent with an element of the crime charged. Here, the is defendant accused of battery, a crime which has no direct connection to a person's truthfulness. While the father would be free to present character evidence that he was a peaceful, non-violent man, evidence regarding his honesty is irrelevant and inadmissible.

Diary offered by sister of victim (out of state on vacation) wherein victim had said she was raped. Admissible?

Yes, because the diary is not being offered to prove that the rape occurred, but to rebut the defense's theory that the victim fabricated the rape claim as revenge. Discussion of correct answer:While the hearsay rule excludes evidence of out-of-court statements offered to prove the truth of the matters asserted therein, the rule does not prevent the admission of such statements offered for other purposes. Here, the victim's sister's diary is not admissible to show that the man raped the victim, but, assuming that it can be properly authenticated, it is admissible as a non-hearsay prior consistent statement to rebut the defense theory that the victim fabricated the rape claim for revenge. While the hearsay rule excludes evidence of out-of-court statements offered to prove the truth of the matters asserted therein, the rule does not prevent the admission of such statements offered for other purposes. Here, the victim's sister's diary, regardless of whether it is authenticated by the victim's sister, is not admissible to prove the truth of the matter asserted therein (that is, to show that the man raped the woman). However, the diary, if properly authenticated, is admissible to rebut the defense's theory that the victim fabricated the rape claim for revenge. The diary could be authenticated by someone else (such as the victim) who was familiar with the diary and could authenticate it. Because this answer does not address the purpose for which the evidence may be admitted, it is not the best response.

A witness reports that a week report someone tried breaking into his house smelling like the defendant was said to have by the victim. Admissible?

Yes, because the evidence suggests that the instructor was the person who committed the attempted rape. Discussion of correct answer:Under Federal Rule of Evidence 404(b), the past crimes or other wrongful acts of defendant are admissible as circumstantial proof of an element of the charged crime, such as motive, intent, absence of mistake, identity, or common scheme or plan (sometimes known as the "MIMIC" rule). In a jury trial, the trial judge must carefully consider whether the risk of undue prejudice substantially outweighs the probative value of the past crimes or wrongful conduct evidence. All "MIMIC" character evidence is in the form of specific acts (as opposed to reputation or opinion evidence). Here, the proffered evidence is relevant on the issue of identity. If the instructor is shown to have smelled as if he hadn't showered in months and worn dirty clothes during the alleged attempted break in to the witness's apartment, it tends to make it more likely that he is the person who attempted to rape the victim in the present prosecution. Federal Rule of Evidence 403 allows a court to exclude relevant evidence if its probative value is substantially outweighed by the danger of unfair prejudice; confusion of the issues; misleading the jury; or by considerations of undue delay, waste of time, or needless presentation of cumulative evidence. The trial court has very broad discretion in applying this standard. Accordingly, this answer choice misstates the above test. In addition, it is unlikely that the prejudice arising from admission of the instructor's other crime would be regarded as substantially outweighing its probative value, since the probative value is strong.

Mother of def brought in to say what her child told her about accident. This is in Federal court, and child raises parent-child privilege. Is state law applicable?

Yes, because the federal court must apply the privilege rules of the state in which it sits. Discussion of correct answer:A federal court sitting in diversity must determine which laws are applicable to the case before it. Generally speaking, the federal court must apply the law of the state in which the court sits as to substantive issues. Federal law is to be applied as to procedural issues. Since evidence law constitutes procedural law, the Federal Rules of Evidence (FRE) generally apply in federal court. However, under some circumstances, rules of evidence may have a substantive effect on a case and affect its outcome. For example, the existence of a particular testimonial privilege may have a substantive effect on a case. Because of the significance of testimonial privileges, although the Federal Rules of Evidence do not have any provisions addressing specific privileges, FRE 501 provides that in civil actions and proceedings, where an element of a claim or defense is not based on a federal question, testimonial privileges are determined by state law. Thus, in an action in federal court based on diversity jurisdiction, the law of the state in which the federal court sits will apply with regard to privileges. Therefore, under the facts presented, the federal court must apply the privilege rules of the state in which it sits. Incorrect. No, because the federal court must apply the federal common law related to privileges. The federal court must apply the federal common law of privileges in criminal cases and federal question cases, but not in civil actions and proceedings in diversity. FRE 501 provides that in civil actions and proceedings, with respect to an element of a claim or defense governed by state law, privilege shall be determined in accordance with state law.

Store manager seeks to use inventory of store to mark what was stolen for cert. Is this allowed?

Yes, because the inventory records were made and kept by the business for just such a circumstance. Discussion of correct answer:FRE 803(6) makes an exception to the hearsay rule for records made and kept in the ordinary course of business. Here, the inventory records were made and kept by the store to track the items on the store's shelves. They are classic business records. Incorrect. Yes, because the inventory records are past recollection recorded. This answer is tempting, but incorrect. Past recollection only comes into play when a testifying witness is unable to remember some fact. Here, the manager never testified that he could not remember something.

The Patient plaintiff wants to stop doctor from mentioning conversation she had with him while her friend was present. May Prosecution bring forth the testimony?

Yes, because the patient's friend was present when the statements were made. Discussion of correct answer:Most states (and, as noted in the facts, the jurisdiction in question here) protect confidential communications made by a patient to a physician for the purpose of obtaining medical treatment. The patient is the holder of the privilege, but the physician may assert the privilege on the patient's behalf. Certain exceptions do exist, however. Most jurisdictions do not apply this privilege: (1) if the patient's physical condition or competency is in issue; (2) in cases where the patient is suing the physician for malpractice; or (3) where the communication was made to assist in the commission of a future crime or fraud. As a general rule, if the communication is overheard by a third party, the confidentiality--and the privilege--are destroyed. The presence of the patient's friend would be the best argument for the inapplicability of the privilege, as the presence of a third party would suggest that the communications made by the patient were not confidential. Incorrect. Yes, because the statements were made for the purposes of the patient's medical treatment. Federal Rule of Evidence 803(4) creates an exception to the general rule excluding hearsay evidence for statements made to physicians for purposes of obtaining medical treatment. However, although the testimony would fall within this exception, the doctor-patient privilege, if applicable, would still bar the statements from coming into evidence. The patient is the holder of that privilege. Thus, if the patient continues to assert the privilege, the testimony will not be admissible, regardless of whether it falls within the exception for statements made for purposes of obtaining medical treatment.

Niece intends to testify that aunt offered her $500 if she stops smoking for 6 months. Admissible?

Yes, because the statement is a verbal act of independent legal significance. Discussion of correct answer:Pursuant to Federal Rule of Evidence 801, hearsay is an out-of-court statement offered to prove the truth of the matter asserted in the statement. Thus, not every out-of-court statement a party seeks to enter into evidence is hearsay. If the statement is offered for a purpose other than proving the truth of the matter asserted in the statement, it is not hearsay. For instance, some statements are the equivalent of verbal acts, as they contain operative facts that have a legal significance independent of the truth of the words contained in the statement. The purpose for offering such statements into evidence is to prove that they were made, not to prove that they are true. Statements that fall into this category include words of offer and acceptance in the formation of a contract, words indicating the intent to make a gift, or words offered to prove slander or libel in a defamation action. Thus, under the facts presented, the aunt's statement is admissible as a verbal act with legal significance independent of its truth. ursuant to Federal Rule of Evidence 801, hearsay is an out-of-court statement offered to prove the truth of the matter asserted in the statement. Thus, not every out-of-court statement a party seeks to enter into evidence is hearsay. If the statement is offered for a purpose other than proving the truth of the matter asserted in the statement, it is not hearsay. For instance, some statements are the equivalent of verbal acts, as they contain operative facts that have a legal significance independent of the truth of the words contained in the statement. The purpose for offering such statements into evidence is to prove that they were made, not to prove that they are true. Statements that fall into this category include words of offer and acceptance in the formation of a contract, words indicating the intent to make a gift, or words offered to prove slander or libel in a defamation action. Thus, under the facts presented, the aunt's statement is admissible as a verbal act with legal significance independent of its truth.

Witness asked if she ever saw someone drunk, and then asked if defendant looked drunk. Is this admissible?

Yes, because the testimony is helpful to the jury in determining the defendant's state of sobriety. Discussion of correct answer:FRE 701 provides that lay opinion testimony is admissible if it is rationally based on the witness' perception and is helpful in determining an issue of fact. Here, whether the defendant was intoxicated would be helpful to the jury in determining fault. Because the witness had an opportunity to observe the defendant and was familiar with observing intoxicated people in the past, her testimony would be useful to the jury's deliberations. Incorrect. Yes, because expert testimony about intoxication is not required. This answer is tempting because it is a true statement of the law--expert testimony is not required here. However, it does not address whether a lay opinion is admissible here.

Radio User was brought in by Plaintiff to testify that she heard (possibly) on the radio what could have been the pilot incriminating himself before the crash. Admissible?

Yes, because the time, frequency, and information contained in the transmission are sufficient additional proof that the speaker was the pilot. Discussion of correct answer:The statement made by the pilot is admissible as a statement by a party opponent. However, these facts present the additional problem of "authentication" of the declarant as the pilot. Here, the pilot in the radio transmission identified himself as "Joe Smith." However, on these facts, identification by the speaker alone is most likely not good enough - the listener had no familiarity with the voice in question, it wasn't from a telephone number assigned to the speaker, it wasn't the ordinary place of the speaker, etc. Ultimately, the name identification, without more, would still need corroborating evidence. In addition to the identification, it is the the time, frequency and information contained in the transmission as heard by the listener, that supply sufficient additional proof which would likely cause the Court to permit the testimony of the listener to be admitted. The jury would still be in a position thereafter to give credibility and/or weight to the testimony. Therefore, this is the best answer choice. Incorrect. No, because there is no way to authenticate a radio transmission that was not recorded. An unrecorded radio transmission can be authenticated by voice recognition or by the self-identification of the speaker. Therefore, this is not the best answer.

Witness a police officer who elicited confession that uncle did the sexual abuse against the niece. Is this admissible?

Yes, because the uncle's confession is a statement of a party opponent. Discussion of correct answer:This evidence is admissible against the uncle as a statement by a party-opponent, which is not hearsay under FRE 801. Incorrect. Yes, because past instances of misconduct may be used to impeach a witness. This choice focuses on impeachment and this is not, strictly speaking, an impeachment situation. The evidence shows the defendant committed the acts complained of in the civil suit. This is a statement by a party-opponent, and is therefore non-hearsay (under the Federal Rules of Evidence) that may be used for substantive purposes. Thus, this answer is incorrect.

Before trial, wife talked in preliminary hearings. On onset, she made up with husband and refuses to talk. Can prosecution use preliminary hearing talk?

Yes, because the wife's prior testimony was sworn and the defendant's attorney had an opportunity to cross-examine her. Discussion of correct answer:When a privilege is asserted which renders the witness incapable of giving testimony, then that witness is unavailable for trial. When a witness is unavailable, the former testimony exception allows hearsay evidence to be admitted it it [Fed. R. Evid. 804(b)(1)]: (1) was given as a witness at a trial, hearing, orlawful deposition, whether given during the current proceeding or a different one; and (2) is now offered against a party who had--or, in a civil case, whose predecessor-in-interest had--an opportunity and similar motive to develop it by direct, cross-, or redirect examination. Here, the defendant's attorney had cross-examined the wife at the preliminary hearing, and she is now unavailable because of her claim of privilege. Therefore, her prior sworn testimony would be admissible at trial. Incorrect. No, because the defendant's Confrontation Clause rights would be violated. The defendant's Confrontation Clause rights would not be violated because he had a meaningful opportunity to cross-examine his wife.

Witness to testify that the conductor hit someone at a rally one month before incident where he got into a fistfight with another Protester, the Plaintiff. Sustain objection against this?

Yes, because the witness's testimony constitutes inadmissible character evidence. (Review this because what!?) Discussion of correct answer:Under Federal Rule of Evidence 404(a), evidence of a person's character "is not admissible for the purpose of proving that he acted in conformity therewith on a particular occasion." The facts here do not show that this testimony would be offered for any other reason (such as the MIMIC reasons of motive, intent, absence of mistake, identity, or common plan or scheme). As such, the evidence constitutes inadmissible character evidence. Incorrect. No, because the witness's testimony is relevant to a material issue in the case. The conductor's propensity for violence is not a material issue in the case. In a civil action for damages arising from a physical attack, the defendant's character for violence would be relevant only to show that he acted in conformity with that character on the occasion in question, which is a prohibited use of character evidence.

Plaintiff committed perjury 7 years ago - admissible?

Yes, because this kind of extrinsic evidence may be offered to impeach the defendant's veracity. Discussion of correct answer:The Federal Rules of Evidence permit the admission of prior convictions for purposes of impeaching any witness if dishonesty or false statements were an element of the crime involved. False statements are an element of perjury charges. If 10 or more years have passed since the later of the conviction or witness's release from confinement for that conviction, then the conviction is only admissible if the court determines the evidence is more probative than prejudicial. In this case, the conviction is only seven years old. Therefore, the evidence should be admitted for purposes of impeaching the defendant's testimony. It is not because criminal convictions within the previous 10 years are admissible (this is wrong). This answer choice is tempting but technically inaccurate. Criminal convictions are not always admissible, even if they occurred within the past 10 years. Not just any criminal conviction can be admitted. Criminal convictions involving dishonesty or false statements are admissible to impeach the credibility of a witness. Convictions that do not involve dishonesty or false statements may not be admissible unless they were punishable by death or imprisonment for more than one year, and the court determines the probative value of the evidence outweighs its prejudicial effect. In this case, the perjury conviction does involve dishonesty or false statements and will, therefore, be admissible on those grounds, not simply because the conviction is less than 10 years old. Therefore, this answer choice is incorrect.

When someone loses the "book", is testimony on that lost evidence allowed and if so why?

Yes, but the weight of the testimony should be left to the jury to evaluate. Discussion of correct answer:Relevant evidence is generally admissible, and, once admitted, it is up to the jury to determine how much weight to give it. The judge need not determine that a cover-up existed to admit the evidence. The jury can decide whether the apprentice's testimony supports an inference of misconduct by the mechanic. Therefore, this is the best answer.

Prior inconsistent statements offered for impeachment purposes only and not to prove the truth of the matter asserted are not hearsay

Yes, for the purpose of impeachment only. Discussion of correct answer:This question presents a rather unusual impeachment situation. What is being impeached here is not the pedestrian who testified, but the bystander, the declarant of the statement about the car running the red light, which the pedestrian repeats in court. That bystander's statement is admissible under the present sense impression exception to the hearsay rule because it was made immediately after the accident and describes the event. The Federal Rules of Evidence allow a party to impeach such a hearsay declarant. The inconsistent statement that the bystander made the day after the accident, which the witness testifies about, is an inconsistent statement admissible to impeach the bystander's present sense impression (testified to by the pedestrian). Therefore, this answer is correct. The testimony concerning the bystander's inconsistent statement is not hearsay. Prior inconsistent statements offered for impeachment purposes only and not to prove the truth of the matter asserted are not hearsay. Therefore, this answer is incorrect.

Police Detective identifies watch noted in a larceny with details of the watch's history (how it got its marks)

Yes, the watch may be authenticated based upon a distinctive marking. Discussion of correct answer:After the relevancy of a given piece of non-testimonial evidence is determined, it must be authenticated prior to being admitted into evidence. Authentication takes place when a witness lays a foundation which is sufficient for a finding that the object is what it is purported to be. The person who seeks to authenticate a physical object must be able to provide testimony which demonstrates that person's personal knowledge of the object, be able to describe distinctive characteristics or markings on the object, or be able to establish that the object is what it purports to be because a chain of custody has been documented and was never broken. In this instance, the officer is being asked to authenticate that the watch is the same as the one he found in the babysitter's home. He does so by pointing to the pry mark, a distinctive marking, and therefore the watch will be admitted into evidence. It is true that the officer has no personal knowledge of the watch that the couple owned. The officer, however, is not being asked to authenticate that the watch to him is the same as the one the couple owned. He is being asked to authenticate that the watch is the same as the one he found in the babysitter's home. Since he has personal knowledge of what he found in the babysitter's home, the authentication cannot be rendered improper on these grounds.

Veterinarian spoke of period occurrences of such dogs attacking, to prove conduct in conformity therewith. Can Defendant object to this testimony?

Yes, this is an improper use of character evidence. Discussion of correct answer:There are general rules which apply to the admission of all evidence and special rules which apply to the admissibility of specific types of evidence such as evidence of subsequent remedial measures, habit evidence, and offers to compromise. Character evidence is also evidence which is subject to special rules governing admissibility. Generally, when we think of character evidence, we think of that character evidence as it pertains to the character of a human. However, character evidence can also be introduced as it pertains to animals in certain circumstances. Specific instances of animal behavior are generally admissible to prove conduct in conformity therewith. So, evidence that a specific animal has previously done a specific bad act can be admitted to show that the animal would be likely to commit that same bad act again. Here, if the veterinarian was being called to testify about an incident in which the resident's dog has bad been violent in the past, then this type of character evidence would be perfectly admissible. However, under these facts, the veterinarian is being called to testify about specific incidences of bad animal acts which were committed by other animals. This is an improper manner of introducing evidence relating to the resident's dog if being offered to prove conduct in conformity therewith. Therefore, it would be proper for the judge to sustain the objection and exclude the veterinarian's testimony for this purpose. Incorrect. No, this is not being offered to prove the truth of the matter asserted. It is a conclusion which is unsupported by the facts presented in the fact pattern. It is stated in the pattern that the evidence of specific bad animal acts is being offered to prove conduct in conformity therewith. That would go directly to the ultimate issue--whether the resident's dog committed this violent act. Therefore, this answer is incorrect because the facts show that the veterinarian's statements are actually being elicited to prove the essential elements of the case.

Defendant was bald at trial, jailer proposes to testify he had thick black hair, just like a witness said, when admitted to prison. Admissible?

Yes, to explain the discrepancy between the defendant's present appearance and the eyewitness's testimony. Discussion of correct answer:It is important to note that the Fifth Amendment applies only to testimonial evidence by the defendant, and a person's appearance, (or fingerprints, etc.) are non-testimonial evidence. Choice (A) is incorrect. A prior identification is an out-of-court statement identifying the defendant. Here, the jailor's identification is being made in court. Choice (C) is incorrect. The testimony about the defendant's prior appearance is highly relevant to help the jury decide the identity of the robber. Choice (D) is inaccurate. There is no reason to exclude this evidence under the Fifth Amendment.

When a defense witness (girlfriend) speaks of defendant's integrity can prosecution bring in other witness to speak against?

Yes, to impeach the driver's girlfriend, and the jury should be so instructed. Discussion of correct answer:In a criminal trial, the prosecution may not offer character evidence to show that the defendant acted in conformity with his character in committing the crime charged. However, if the defendant "opens the door" by introducing evidence of a pertinent character trait to show that he is innocent of the crime charged, the prosecutor may rebut this evidence either by cross-examining the defendant's character witness as to the witness's knowledge of specific instances of the defendant's prior conduct or by calling witnesses to testify as to their opinion of the defendant's character or as to their knowledge of the defendant's reputation in the community. If, however, the defendant did not call a witness with the intent that the witness testify as to the defendant's good character, but the prosecutor elicits such testimony on cross-examination, the prosecutor cannot then (based on the testimony the prosecutor elicited) introduce reputation or opinion evidence of defendant's bad character to show that the defendant acted in conformity with this character. (In other words, the defense itself must "open the door.") However, the prosecutor may introduce evidence that contradicts the witness's testimony for the purpose of impeaching the witness. Thus, here, the teen's testimony regarding the driver's reputation is admissible to impeach the driver's girlfriend. In a criminal trial, the prosecution may not offer evidence of a defendant's character in order to prove that the defendant acted in conformity with his character in committing the crime charged. Thus, here, the testimony would be inadmissible to prove that the driver acted in conformity with his reputation for drug dealing, but the evidence would be admissible for impeachment purposes.

Defendant seeks to introduce evidence a photograph taken of the criminal transaction to show that the officer could not have seen what the officer proposed was the defendant in action. Admissible?

Yes, to impeach the police officer. Discussion of correct answer:One method of impeaching a witness is by sensory defects. The purpose of such impeachment is to show that the witness does not have knowledge of the fact or facts about which they are testifying. This can be done on cross-examination or by using extrinsic evidence. Here, the defense is admitting the photograph to impeach the police officer on his ability to observe the transactions that allegedly took place between the defendant and the buyers.

Plaintiff show order form to refresh memory. May defense lawyer have this admitted to evidence?

Yes, to let the jury compare the contents of the document and the witness's testimony. Discussion of correct answer:A document used by a witness to refresh recollection before testifying must be admitted into evidence if the attorney for the other side wants the trier of fact to see the document to compare it with the testimony. The reason for this is to allow the jury to evaluate whether the witness is just reiterating the contents of the document or is testifying from memory of the events. This may be helpful to the jury in evaluating credibility and deciding how much weight to give the testimony. Choice (A) is a misnomer; the wording mixes up two separate doctrines: 1) present recollection refreshed (applicable here) and 2) past recollection recorded (a hearsay exception applicable when a document made shortly after an event is read into evidence, because the witness is unable to recall the event). Note that in the latter case, the document does not go to the jury. Choices (C) and (D) are incorrect. A document that was used to refresh recollection may be given to the jury to assist them in evaluating credibility, irrespective of the usual rules (i.e., hearsay, best evidence, etc.) affecting admissibility of documents that are offered to prove a substantive point. Incorrect. Yes, as a prior recollection recorded.

Can plaintiff bring in certificate of conviction stemming from same crash wherein def was found guilty of DUI? Is it Admissible?

Yes, to prove an essential fact. Discussion of correct answer:FRE 803(22) allows admission of an otherwise hearsay certificate of conviction if: (1) the conviction is for a felony offense; and (2) it is necessary to prove an essential element in the civil case. Here, the plaintiff's theory is that the defendant was negligent in causing the accident because he was drunk. Evidence in the form of a certificate of conviction is proof that he was intoxicated at the time of the accident, an essential element of the plaintiff's case. Incorrect. Yes, as a public record. This choice is incorrect because the public records exception does not apply. FRE 803(8) allows admission of an otherwise hearsay record or statement of a public office if it sets out: (1) the office's activities; (2) a matter observed while under a legal duty to report, but not including, in a criminal case, a matter observed by law enforcement personnel; or (3) in a civil case or against the government in a criminal case, factual findings from a legally authorized investigation. A certificate of conviction does not fall under any of these categorie

Business man brought in a Aztec statue with drugs. Claims not to have known. 11 years prior he had another statue with drugs. Admissible?

Yes, to rebut the businessman's claimed ignorance of the drug's presence. Discussion of correct answer:Evidence questions are difficult because they often test several different rules of law within the same question. This question deals with the interplay between character and impeachment evidence. Federal Rules of Evidence 404(a)(1) and 404(a)(2) require, in a criminal case, that the defendant open the door with either his good character or victim's bad character. Here, the prosecution is attempting to introduce the evidence, not the defendant, so choice (C) looks good because it states the general rule. Choice (D) also states that the proposed evidence is inadmissible, and cites Rule 609(2) dealing with impeachment. Here, both rules are correctly stated, however, do not account for Rule 404(b), often cited as the MIMIC Rule. Usually in a criminal case, on rebuttal, the prosecution may use otherwise inadmissible character evidence for some purpose other than to show the defendant acted in conformity therewith. Here, the defendant claims ignorance, that he was unaware of the presence of the narcotics inside the sculpture. The prosecution wants to rebut defendant's claimed ignorance with evidence that he smuggled drugs into the country previously, and in a similar fashion. This evidence will be admissible under Rule 404(b) and therefore choice (B) is correct.

Def challenges verdict and calls jurors to testify. Which witness would not be considered competent to testify under FRE?

A juror testifying as to the votes that were taken in reaching the verdict. Discussion of correct answer:Under Rule 606, a juror is not competent to testify regarding the manner in which the jury reached its decision even upon an inquiry into the validity of a verdict or indictment. No juror may testify to any statements made during the deliberations, the thought processes by which the juror arrived at a decision, or votes that were taken. Therefore, this juror would be deemed to be incompetent to testify. Incorrect. The judge who presided over the original trial who is not the judge presiding over the current challenge to the verdict. A judge may never testify as a witness in a trial over which the judge presides. The testifying judge here, however, is not presiding over the current challenge. There is no rule against a judge being called to testify about a case once that case has concluded.

Defendant had told witness at scene of his fault in an excited manner. How may it be admitted?

A statement of an opposing party or an excited utterance. Discussion of correct answer:The engineer's statement immediately following the accident is an out-of-court statement offered to prove the truth of the matter asserted therein--that is, that the engineer did not see the stop sign in time and that he should have been going slower. As such, one must considered whether the hearsay rule bars admission of the statement. Upon analysis, however, it is clear that the statement is admissible on a number of grounds. For one, as a statement made by the engineer while he was still under the stress of a traumatic event, the engineer's statement falls within the excited utterance exception to the hearsay rule. Also, the engineer is a party to the lawsuit and the statement is his own statement, now offered against him at trial. Under the Federal Rules of Evidence, such a statement is classified as admissible nonhearsay. As such, the engineer's statement is admissible under either of these two theories. Incorrect. A statement of an opposing party. It is true that as the engineer is a party to the lawsuit and the statement is his own statement, now offered against him at trial, it is classified under the Federal Rules of Evidence as admissible nonhearsay. However, while the engineer's statement is admissible, this is not the only ground on which it may be admitted. As such, this is not the best answer.

A medical passage contradicts plaintiff expert's opinion. Admissible?

Admissible, as substantive evidence. Discussion of correct answer:Rule 803(18) of the Federal Rules of Evidence provides that statements contained in treatises may be admitted into evidence during direct or cross-examination of an expert witness if: (1) the treatise is established as a reliable authority; and (2) the treatise is called to the attention of the expert witness during cross-examination or is relied upon by the expert in direct testimony. As an exception to the hearsay rule, statements in treatises admitted pursuant to Rule 803(18) may be used as substantive evidence, by reading them to the jury.

Testimony of a witness who is the head veterinarian of a clinic on 5 earlier dogs with similar conditions. Admissible?

Admissible, because it is relevant to a causal connection between the dog's ailment and the medication. Discussion of correct answer:Relevant evidence is evidence that has any tendency to make a fact that is consequential to the litigation more probable or less probable than it would be without the evidence. A civil litigant may attempt to introduce evidence of prior similar incidents (or the lack thereof) as circumstantial evidence to support the proponent's claim or defense. As a general rule, most courts will not admit evidence of prior incidents as proof of a defendant's negligence and will not admit evidence of the lack of prior incidents as proof that a defendant exercised due care. The probative value of such evidence is simply too weak when balanced against the risk of unfair prejudice and a waste of court time. However, if the evidence is offered to prove: (1) that a dangerous condition existed; or (2) that the defendant was aware of the dangerous condition, the evidence will be admissible on such grounds. Here, evidence that five other dogs treated with the same drug had suffered blood clots identical to the dog owner's dog tends to prove that the medication at issue caused the dog's injuries. This causal link is a necessary element of the dog owner's cause of action, and so the evidence is relevant and admissible if there is evidence that the medication caused the same injury to other dogs under circumstances materially similar to the case at hand. It is true that the fact that a witness qualifies as an expert permits him to testify in the form of opinions that would otherwise be inadmissible. Here, however, the witness would be testifying that he treated other dogs with medical conditions similar to the dog owner's dog. Thus, his testimony would not be opinion testimony, but testimony by an eyewitness. Furthermore, even if the witness is a qualified expert, that alone will not render his testimony admissible. To be admissible, expert opinion must be relevant to a material issue in the litigation.

Owner of car tells mother at hospital his insurance should cover the boy's medical bills. Admissible?

Admissible, only to prove ownership or control of the cart that injured plaintiff. Discussion of correct answer:Under Federal Rule of Evidence 411, evidence that a person was or was not insured against liability is not admissible on the issue of whether he acted negligently or with fault. However, the rule does not require the exclusion of evidence of insurance when offered for another purpose, such as proof of ownership or control of the cart. Sometimes such a stipulation is included on the MBE fact patterns, so you should look carefully for such facts. An offer to pay for medicine is improper to use for evidence purposes.

Does not constitute a check adequate authentication of the checks signed?

Comparison of the checks by the police officer who made the arrest with an admittedly genuine specimen of the banker's handwriting. Discussion of correct answer:By process of elimination, choice (D) is the only possible correct answer. Under Rule 901(b)(2) of the Federal Rules of Evidence, a person's handwriting may be authenticated by "non-expert opinion as to the genuineness of the handwriting based on familiarity not acquired for purposes of the litigation." Choice (A) is therefore a correct statement. Similarly, choices (B) and (C) are accurate statements under Rule 901 since authentication may be made by "comparison by the trier of fact or by expert witnesses with specimens which have been authenticated." As a result, choice (D) is the best answer (and thus a false statement) since the arresting officer's familiarity with banker's handwriting was acquired for purposes of prosecution.


Related study sets

PMP Exam Chapter 11 Risk Management

View Set

6 major functions of the cell membrane

View Set

CH 30: ABDOMINAL AND GENITOURINARY INJURIES

View Set

Chapter 10 ‑ THE WESTERN INDIANS

View Set

chapter 6b-health insurance policy provisions

View Set

E.3.1 Use context clues to determine the meaning of words or phrases

View Set